Mental State/Intellectual Function/Behavioural Problems+Major Psychiatric Disorders/Drug & Alcohol Abuse Flashcards
A 20-year-old girl presents with history of episodes of trembling, palpitation, shortness of breath, and sweating. Each episode lasts for 5 minutes and then gradually resolves. She has no history of heart disease, and is not on any regular medication. She denies any use of illicit drugs. Which one of the following would be the best course of action for management?
A. Reassurance, explanation, and support.
B. Selective serotonin reuptake inhibitors.
C. Breathing in and out of a paper bag when attacks develop.
D. Tricyclic antidepressants.
E. Referral to a psychiatrist.
Correct Answer Is A.
Panic Attack:
- Definition: A panic attack is a sudden onset of intense fear or discomfort that peaks within minutes and includes a variety of physical and cognitive symptoms.
- Symptoms: These can include palpitations, sweating, trembling, shortness of breath, chest pain, nausea, dizziness, chills, hot flashes, numbness, derealization, depersonalization, fear of losing control, and fear of dying.
- Duration: Typically peaks within 10 minutes and can last about 20-30 minutes.
- Context: Panic attacks can occur in various situations, including during stressful events, within the context of other anxiety disorders, or independently.
Panic Disorder:
- Definition: Panic disorder is diagnosed when a person experiences recurrent unexpected panic attacks and has ongoing concern or worry about having more attacks or their consequences (such as losing control or having a heart attack).
- Criteria:
- Recurrent unexpected panic attacks.
- At least one of the attacks has been followed by one month (or more) of one or both of the following:
- Persistent concern or worry about additional panic attacks or their consequences.
- A significant maladaptive change in behavior related to the attacks (e.g., behaviors designed to avoid having panic attacks, such as avoidance of exercise or unfamiliar situations).
- Impact: The disorder leads to significant distress or impairment in social, occupational, or other important areas of functioning.
- Management: Treatment often includes cognitive-behavioral therapy (CBT) and may include pharmacotherapy, such as selective serotonin reuptake inhibitors (SSRIs) or benzodiazepines.
Key Differences:
- Frequency and Impact: Panic attacks can occur as isolated incidents, whereas panic disorder involves repeated attacks and significant ongoing worry and behavioral changes.
- Diagnosis: Panic disorder requires recurrent unexpected panic attacks and persistent concern or behavioral changes due to the attacks, impacting daily life.
For more detailed information, consult RACGP guidelines or resources specific to panic disorder management.
This patient has typical presentation of panic attacks.
Panic attack is characterized by the presence of at least 4 of the following features:
* Palpitations, pounding heart, or accelerated heart rate
* Sweating
* Trembling or shaking
* Sense of shortness of breath or smothering
* Feeling of choking
* Chest pain or discomfort
* Nausea or abdominal distress
* Feeling dizzy, unsteady, lightheaded, or faint
* Derealization or depersonalization (feeling detached from oneself)
* Fear of losing control or going crazy
* Fear of dying
* Numbness or tingling sensations
* Chills or hot flushes
The cornerstone of treating panic attacks is explaining the condition to the patient, reassurance and supporting the patient to understand the situation. For residual anxiety following a panic attack
benzodiazepines might be considered.
(Options B and D) Selective serotonin re-uptake inhibitors (SSRIs) and tricyclic antidepressants (TCAs) have been used as pharmacological treatment of panic disorder (not panic attacks) as complements to nonpharmacological management (e.g. CBT), which is first-line treatment of panic disorder.
(Option C) Breathing in and out of paper bag is helpful if patient is hyperventilating during a panic
attack. It can provide relief in short time and can be taught to the patient after explanation and reassurance.
(Option E) Referring to a psychiatrist is not needed at this stage because the diagnosis is completely straightforward and clear.
An 82-year-old man male presents to the emergency department with a knife in his hand. He claims that he hears voices saying his neighbors want to kill him and he should kill them before they kill him. You, as the attending physician, manage to verbally de-escalate him and convince him to surrender his knife. Which one of the following would be the next best step in management?
A. Call the hospital security.
B. Call the police.
C. Call his family to help you with dealing with him.
D. Calm him down and talk to him.
E. Offer him tea and biscuit.
Correct Answer Is A.
The presentation is typical of acute psychosis and command hallucinations. Any patient with command hallucinations to harm self or others is at significant risk of developing violence sooner or later.
Although apparently this patient is ready to surrender his knife now, with command hallucinations he may still pose risk to himself or others.
Under circumstances such as this one, calling the hospital security would be the next best step in
management. This will ensure the safety of you and the staff while you are planning further measures.
(Option B) Calling the police was an appropriate option if this situation happened in the community
and not in a medical facility.
(Option C) Calling his family will provide an opportunity to obtain more information about his
medical and psychiatric history; however, security and safety comes first.
(Options D and E) Verbal de-escalation to calm the patient or measures to establish rapport such as offering tea and biscuits is appropriate after safety is ensured by the hospital security.
John, 24 years, had a diagnosis of depression and was started on sertraline (Zoloft) 50 mg/day, 23 days ago, and was given a mental health care plan to see a psychologist for cognitive behavioral therapy (CBT). He is in your office today for follow-up, and believes there has been no improvement whatsoever. Which one of the following is the most appropriate** next step** in management?
A. Continue the same dose of sertraline.
B. Increase the dose of sertraline.
C. Switch to another SSRI.
D. Switch to an SNRI.
E. Add a different antidepressant medication to sertraline.
Correct Answer Is A.
Selective serotonin reuptake inhibitors (SSRIs) and tricyclic antidepressants (TCAs) are the two most prescribed medications for major depression.
Other major drug classes include serotonin norepinephrine reuptake inhibitors (SNRIs), (e.g.,
venlafaxine), reversible monoamine oxidase inhibitors (e.g., moclobemide), and non-selective monoamine oxidase inhibitors (e.g., phenelzine).
SSRIs have safer adverse effect profile and less likely to cause dropout compared to TCAs.
Antidepressant are started at initial dose and gradually and incrementally increased in dosed until
therapeutic doses are reached. For most SSRIs the initial dose is the same as therapeutic dose. As such, 50mg per day is both the initial and therapeutic dose for sertraline. The therapeutic response to therapeutic dose is often delayed for 1-2 weeks, and as a rule of thumb, all antidepressants should be trialed for at least 4-6 weeks after the therapeutic dose is reached before any change in treatment is considered.
John has been started on sertraline 20 days ago and it is still early to consider a change to treatment such as increasing the dose of sertraline (option B), switching to another SSRI (option C) or an SNRI (option D), unless for adverse effects issues which feel bothersome to the patient. John should be advised to continue the same dose for at least to 6 weeks under close monitoring for any complications (e.g., adverse effects, suicidality, etc.)
Using two antidepressants together is NOT recommended. Therefore, addition of another
antidepressant to sertraline (option E) is not a correct option.
Janet presents to you concerned about her 15-year-old son, Joey, after she found out he had been
wearing his older sister’s underwear. She is a single parent as her husband left them after Joey was born. She works two jobs and has been under a lot of stress and has not had enough time to spend with her kids. She is quite frustrated and distressed with Joey’s behaviour and believes there is something seriously wrong with him and wishes she has not given birth to him at all. Which of the following is the correct statement to share with Janet?
A. Joey may feel as if he is a girl trapped in a boy’s body.
B. He is most likely doing it for fun.
C. Not having a father figure in his life is the reason for this behavior.
D. Janet’s lifestyle may be the main reason for Joey’s behavior.
E. It is a part of normal development at this age.
Correct Answer Is A.
Joey’s interest in wearing clothing suggests differential diagnoses such as cross dressing, transvestic disorder, gender dysphoria, and transgenderism. All these diagnoses should be born in mind, discussed with Janet, and assessed thoroughly.
It is normal for children and teenagers to experiment with gender. For example, a girl might refuse to wear skirts or dresses, or a boy wants to play ‘mum’. For most children and teenagers, experimenting with gender does not mean they are gender diverse or transgender. Most children go on to feel comfortable with their birth gender. In contrast, gender dysphoria in children and adolescents is not a phase. Gender dysphoria is when one feels distressed because their gender identity differs from the birth sex. This distress might affect their school or home life. Those with gender dysphoria feel like they are trapped in a body of the opposite sex. It should be explained to
Janet while going over potential diagnoses and how one would feel if they were gender dysphoric or transgender.
Janet is quite worried about her son and his behavior; therefore, telling her that he is most likely doing this for fun (Option B) would not be reassuring to her, especially when you do not have all the facts to draw such conclusion. It would be more prudent to counsel her and explain to her Joey may be experiencing and encourage her to bring him in for counselling if he wants it. Personal counselling and family insight therapy should be recommended.
There is not enough information in the scenario to suggest that not having a father figure in his life
(option C) or Janet’s lifestyle (option D) could be the reason for Joey’s manner. Although both
these reasons are disruptive to the normal functioning of their family life, Joey does not exhibit any other characteristics or behaviors to suggest that he is acting out in response to these disruptions.
By about the age 6 or 7 years, children begin to understand that sex is permanent across situations
and over time. Once they develop this understanding, they begin to act as members of their sex. Therefore, it would be inaccurate to tell Janet that Joey’s behavior is a part of normal development (Option E) at the age 15.
TOPIC REVIEW
Cross-dressing/ transvestitism
Cross-dressing is defined as typically heterosexual men wearing women’s clothing. There is no correlation between crossdressing and transgenderism or homosexuality. Cross dressers choose to dress as women only some of the time and enjoy experiencing both the masculine and feminine
parts of themselves. As opposed to transvestitism, a transgender person lives fulltime in the gender that they identify with.
DSM-5 considers cross-dressing as a psychiatric disorder (transvestic disorder) if cross-dressing
or thoughts of cross-dressing are always or often accompanied by sexual excitement. The main difference between cross-dressing and transvestitism is that the latter is associated with sexual arousal.
Transgenderism
Children whose gender identity differs from the gender they were assigned at birth are known as
transgender or gender diverse. Transgender is a non-medical term describing individuals whose gender identity (inner sense of gender) or gender expression (outward performance of gender) differs from the sex or gender to which they were assigned at birth. Not all transgender people suffer from gender dysphoria and that distinction is important to keep in mind. Gender dysphoria and/or coming out as transgender can occur at any age.
People who are transgender may pursue multiple domains of gender affirmation, including social
affirmation (e.g., changing one’s name and pronouns), legal affirmation (e.g., changing gender markers on one’s government-issued documents), medical affirmation (e.g., pubertal suppression or gender-affirming hormones), and/or surgical affirmation (e.g., vaginoplasty, facial feminization surgery, breast augmentation, masculine chest reconstruction, etc.). Of note, not all people who are transgender will desire all domains of gender affirmation, as these are highly personal and individual decisions.
Eve is 45 years old and has presented to your clinic for consultation in tears. Her only son died 6 weeks ago from a car accident, and since then she has been having difficulty sleeping and terrible headaches. She does not enjoy doing her life routines anymore and does not feel like eating.
Further probing reveals that she hears her son speaking to her and calling her name and sometimes she feels his presence in her bedroom at night. She wants something to help her with her sleep. Which one of the following is the most appropriate action in this situation?
A. Referral to hospital for admission, assessment, and treatment.
B. Referral to a psychiatrist.
C. A short course of benzodiazepines and referral for grief counselling.
D. A short course of selective serotonin re-uptake inhibitors (SSRI) and referral for grief
counselling.
E. A short course of antipsychotics and referral for grief counselling.
Correct Answer Is C.
Given the symptoms and their duration, a normal grief reaction is the most likely explanation to this woman’s problem. Janet is grieving for her deceased son, and this has affected her in many ways, including a disruption of her daily routine, weight loss and decreased sleep and appetite. Patients with normal grief reaction return to normal social functioning within 2 months. The symptoms however might last up to one year with waxing and waning.
Eve has come to your clinic seeking help; in particular, she wants something to help her with her sleep. In the absence of any red flags (suicidality, drug seeking behaviour, etc.), the most appropriate action would be to prescribe a short course of benzodiazepines (< 7 days) for her insomnia and referral to grief counselling to help her cope with her loss.
Sadness, despair, tearfulness, decreased sleep, decreased appetite and decreased interest in life
and the world, are some of the common findings in normal grief reaction. While guilt and shame are not common in normal grief reaction, they are still possible. Suicidality is not usually a concern unless the patient has said something indicative of her intent and plans to do so which is not the case here.
Eve might require referral to psychiatrist (option B) down the line if grief counselling does not
resolve the issue and symptoms persist. It is not necessary for now.
SSRIs would be indicated if Eve had a diagnosis of moderate to severe depression. Eve does not fulfill criteria for a diagnosis of major depression. While referral for grief counselling is necessary, SSRIs are not indicated neither for depressive symptoms nor for the sleep issue.
Simple visual and auditory hallucinations of the deceased person are common and may lead the bereaved person to fear he/she is losing their mind. These are a result of her grief reaction, and not new onset psychosis. Similarly, Eve is hearing her son speaking to her and calling her name and sometimes, she feels his presence in her bedroom at night. This could be considered normal in grief as long as there are no other psychotic features necessitating antipsychotic treatment. Eve does not have psychotic illness and antipsychotics (option E) would not be indicated.
A middle-age famer from Queensland presents with symptoms of poor sleep, lack of concentration and energy, decreased appetite, and low mood for the past few weeks. He believes that all these started after the drought and blames the government for being reckless and irresponsible. When you mention that his symptoms are of depressive nature, he objects it and says he is just exhausted and not depressed. Which one the following defense mechanisms is he using?
A. Displacement.
B. Projection.
C. Denial.
D. Rationalization.
E. Reaction formation.
Correct Answer Is C.
This patient is experiencing symptoms of depression (poor sleep, lack of concentration and energy, decreased appetite, and low mood) brought on by his recent losses after the drought. As the doctor attempts to explain to him that his symptoms are of a depressive nature, he refuses to accept it and insists that his symptoms are a result of exhaustion and not depression. Based on the facts in the scenario, this patient is most likely utilizing denial as a defense mechanism. Denial is a form of psychotic defense mechanism evident by the replacement of external reality with wishful fantasy (behaving as if an aspect of reality does not exist).
When counselled by the doctor, he said he blamed the government for being reckless and irresponsible. This is an example of projection (Option B), another form of a defense mechanism.
Projection refers to the interpretation of internal impulses as though they are outside oneself (attributing one’s own feelings to others). In this scenario, the drought could have been a cause for his symptoms as it might have affected his livelihood as a farmer, but the patient places blame on the government without accepting his share of responsibility to have taken adequate safety
measures in case of natural causes. Displacing his feelings helps him subconsciously place the root cause of this feeling onto someone or something else. In other words, he is projecting the negative feelings of recklessness and irresponsibility elsewhere to protect his ego. However, this question refers to the patient’s reaction to being informed that his symptoms are of a depressive nature. Therefore, projection (option B) is not the correct option.
Displacement (option A), another defense mechanism and occurs when a person represses
affection, fear, or impulses that they feel towards another person as they believe it is irrational or socially unacceptable to demonstrate such feelings; therefore, these feelings are displaced toward another person or thing.
Rationalization (option D) defense mechanism involves using rational explanations to justify behaviors that are unacceptable (justifying behaviour to avoid difficult truths). In doing so, they avoid accepting the true cause or reason resulting in the present situation. This patient would be using rationalization if, for example, he said: “of course I am depressed. The drought has destroyed all my crops and left me with losses”. In this scenario, however, he is denying his depressive symptoms altogether.
In reaction formation (option E) an individual expresses the opposite of their true feelings, sometimes to an exaggerated extent. It is an intentional effort to compensate for conscious dislikes. For example, if he felt that his self-esteem was threatened by being diagnosed with depression, he would have acted overly aggressive. This is not the case here, and therefore reaction formation is not the correct option.
Ali, 45 years old is in your office with his wife who is concerned about him and insisted that he sees a doctor because he seems to have been ‘lost’ and ‘confused’ since he had a severe car accident and lost one of his best friends who was in the car with him 5 months ago. He only sustained a head injury in the accident which was cleared as a minor one with no serious complications at that time. Which of the following could be the most likely cause to this presentation?
A. Major depressive disorder.
B. Post-traumatic stress disorder.
C. Post-concussion syndrome.
D. Late-onset schizophrenia.
E. Dementia.
Correct Answer Is C.
This Scenario represents a common undiagnosed and mistreated condition seen in general practice after a head trauma, namely post-concussion syndrome (PCS).
Concussion is a temporary disturbance in brain function following a trauma to the head. It can also
occur after a blow to the body. Concussion can present with a variety of signs and symptoms
including:
* Cognitive related symptoms:
* Difficulty concentrating
* Difficulty finding things
* Difficulty reading
* Memory problems
* Brain fog
* Easily distracted
* Mood-related symptoms:
* Anxiety
* Depression
* Feeling overwhelmed
* Irritability
* Low-energy or motivation
* Various other mood/personality changes
* Sensory-related symptoms:
* Blurred vision
* Car sickness or nausea with motion
* Change in (or loss of) taste or smell
* Ringing ears
* Blood dysregulation symptoms:
* Headache
* Fatigue
* Nausea
* Dizziness
* Sensitivity to light and noise
* Sleep problems
* Persistent neck pain
* Pressure in the head
* Tried eyes
Almost 90% of patients with concussive symptoms experience a quick recovery within few days to weeks. Those in whom symptoms persist beyond 3 moths are defined as having post-concussion syndrome (PCS).
Since Ali has been in a car accident resulting in his friend’s death, post-traumatic stress disorder (PTSD) (option B) should be considered as well. Symptoms may include flashbacks, nightmares, and severe anxiety, as well as uncontrollable thoughts about the event. Symptoms should persist for at least 1 month before a diagnosis of PTSD is made. The DSM-5 has a more expansive set of criteria including 20 different symptoms across the domains of re-experiencing, avoidance, negative cognitions and moods, and hyperarousal. Ali has none of such symptoms; therefore, unlikely to have PTSD.
Major depressive disorder (option A) requires the persistence of 5 or more of the 9 Criteria A symptoms listed in the Diagnostic and Statistical Manual of Mental Disorders, 5th edition (DSM-5) – depression, loss of pleasure, weight change, sleep change, retardation, loss of energy, feelings of worthlessness, diminished concentration, thoughts of death – in a person who has never had an episode of mania. Ali does not meet the criteria for major depressive disorder.
Late-onset schizophrenia (option D) is defined as schizophrenia starting after the age of 45 years.
Patients with late-onset schizophrenia typically present with the same positive psychotic symptoms (paranoid delusions and hallucinations) as do younger schizophrenic patients, although these symptoms are less severe in older patients. In contrast with early-onset disease, disorganised thoughts and negative symptoms (flat affect, alogia, avolition) are less likely to occur in patients with late-onset disease. Late-onset schizophrenia is also less likely to be associated with impaired learning and cognitive functions. Ali has no paranoia, delusions or hallucinations; therefore, not likely to have late-onset schizophrenia.
Dementia is a clinical syndrome that is caused by a number of underlying diseases. Such as vascular dementia, frontotemporal dementia, dementia with Lewy body, Alzheimer’s. The DSM-5 diagnostic criteria for dementia include the following:
* Significant cognitive decline from a previous level of performance in one or more cognitive domains (i.e., complex attention, executive function, learning and memory, language,perceptual-motor or social cognition.
* The cognitive deficits interfere with independence in everyday activities (paying bills, managing medications).
* The cognitive deficits are not better explained by another mental disorder (e.g., major depressive disorder, schizophrenia).
Ali has no memory loss, language problem, or cognitive deficit; therefore, unlikely to have dementia
(option E).
During taking a history from a patient in a psychiatry ward, you ask him if there is any history of mental illnesses in the family. He answers: ‘Good question. Now that you’ve asked, I must tell you something. Whales are good creatures. We take their oil and use it to light the world. My father hunted whales as did my grandpa. The business runs in the family and of course, we are all sane and sound. Do you think we are crazy?” Which one of the following is present in this patient’s speech?
A. Flight of ideas.
B. Word salad.
C. Circumstantiality.
D. Derailment.
E. Tangentiality.
Correct Answer Is C.
The speech given by this patient as the answer to the question of whether there is a family history of mental illnesses starts with a comment on an unrelated topic, whales. Thereafter, each next sentence uses a clue in the previous sentence for continuation. At the end and after many detours the answer to the question is provided: ‘We are all sane and sound’ (meaning there is no family history of mental illness and they all have been sane and sound.
This pattern fits ‘circumstantiality’ best.
Circumstantiality occurs when the patient drifts from one topic to the other but eventually returns to the starting point. In other words, if a question is imagined as a destination, there are many detours, but the destination is reached at the end. Unlike
circumstantiality, patients with derailment (option D) (loosening of association) never come back
to the topic they started off with. Tangentiality (option E) or tangential speech is a milder form of derailment in which there is a hint linking two consecutive topics (whales and the whale oil that
light up the world [whale oil in the past had industrial use including for lighting and as fuel]).
Flight of ideas (option A) is characterized by over-productive speech with rapid shifting from one
topic to another. There is often a hint in the previous topic leading to the next one. In the flight of ideas, there is a subjective feeling that the thoughts are racing. In the flight of ideas, the topic spoken by the patient is organized but over-productive and in excess of details. In other words, the general concept of the current topic is adequately understood, but a hint in one part leads the patient to another. This patient follows a direct line made of pieces that are relevant and justify his sanity and that of his family for that matter, rather than jumping from one topic to another.
In word salad (option B), the patient throws words together without any sensible and intelligible
meaning.
Jarred, 15 years old, is brought to you by his concerned parents for assessment. According to them, he recently has significantly declined school performance. He is quite withdrawn and barely
leaves his room. He does not engage in social activities and even rarely talks with his parents. Which one of the following, if present in the history of the mental exam, is most likely to help reach a diagnosis?
A. Sleep issues.
B. Anxiety.
C. Loosening of association.
D. Use of recreational drugs.
E. Shyness.
Correct Answer Is C.
Scenario Overview
- Presentation: Social withdrawal, minimal interaction with parents, decline in school performance.
- Possible Causes: Mood disorder or prodromal phase of early-onset schizophrenia.
- Key Symptom: Loosened association in speech, suggesting early-onset schizophrenia.
Loosened Associations
- Definition: Lack of logical connection between ideas in speech.
- Example: “I like to dance; my feet are wet.”
- Significance: Common in schizophrenia, indicative of psychotic disorders.
Symptoms of Early-Onset Schizophrenia
- Thinking:
- Problems with reasoning
- Bizarre thoughts or speech
- Confusion between dreams and reality
-
Behavior:
- Social withdrawal
- Sleep disturbances
- Reduced motivation (e.g., decline in school performance)
- Neglect of personal hygiene
- Bizarre or aggressive behavior
- Substance use
-
Emotions:
- Irritability or depression
- Inappropriate emotions
- Unusual fears or suspicions
Diagnostic Considerations
- Sleep Issues (Option A): Non-specific, can occur in various mental conditions.
- Anxiety (Option B): Not consistent with other symptoms, complicates diagnosis.
- Recreational Drug Use (Option D): May exacerbate or mimic psychiatric symptoms.
- Shyness (Option E): Normal behavior, not diagnostically relevant.
DSM-5 Criteria for Schizophrenia
- Symptoms: Delusions, hallucinations, disorganized speech, grossly disorganized behavior, negative symptoms.
- Duration: Symptoms persist for ≥ 1 month, impairment lasts ≥ 6 months.
- Exclusion: Other causes for symptoms must be ruled out.
Terminology
- Childhood-Onset Schizophrenia: Before age 13.
- Early-Onset Schizophrenia: Before age 18.
- Adult-Onset Schizophrenia: At or after age 18.
Summary
Early-onset schizophrenia presents with social withdrawal, cognitive disturbances, and altered behavior. Loosened associations in speech are indicative. Understanding these symptoms and diagnostic criteria helps in early recognition and intervention.
The scenario illustrates social withdrawal represented by not engaging in social activities, barely talking with the parents, and not leaving the room. He also has a significant decline in school performance. This constellation of problems is likely to have been caused either by a mood disorder or by the prodromal phase of early-onset schizophrenia. Of the options, the presence of loosened association is highly predictive and suggestive of the latter.
“Loose associations” is a psychological term to describe a lack of connection between ideas. This
can manifest in speech as an individual moving quickly from one idea to an unrelated one in the same sentence, expressing a random jumble of words and phrases. An example of a loose association would be: “I like to dance; my feet are wet.” Loosening of association is a key symptom in psychotic disorders such as schizophrenia.
Schizophrenia typically manifests with a prodrome of negative symptoms and psychosis (e.g., social withdrawal) that precedes the positive psychotic symptoms (e.g., hallucinations and bizarre delusions).
Childhood-onset and early-onset schizophrenia are more severe and debilitating forms of schizophrenia. Early signs and symptoms may include problems with thinking, behavior, and emotions:
Thinking:
* Problems with thinking and reasoning
* Bizarre ideas or speech
* Confusing dreams or television for reality
Behavior:
* Withdrawal from friends and family
* Trouble sleeping
* Lack of motivation e.g., presenting with a drop in performance at school
* Not meeting daily expectations, such as bathing or dressing
* Bizarre behavior
* Violent or aggressive behavior or agitation
* Recreational drug or nicotine use
Emotions:
* Irritability or depressed mood
* Lack of emotion, or emotions inappropriate for the situation
* Strange anxieties and fears
* Excessive suspicion of others
Sleep issues (option A) could be present in schizophrenia and other psychotic disorders as well as a wide variety of other mental conditions such as mood disorders or anxiety disorders. In and of itself, sleep issues are neither specific nor diagnostic.
Anxiety (option B) may indicate anxiety disorders; however, other symptoms are not consistent with anxiety disorders. Therefore, its presence would not help with justifying this clinical
presentation and in fact will add more of a diagnostic challenge.
Individuals with mental illnesses are more prone to using recreational drugs. Also, the use of recreational drugs may lead to mental issues (e.g., acute psychosis, withdrawal syndromes, etc.) If there is a history of recreational drug use in Jarred (option D), it could be a behavioral issue associated with a mental illness such as schizophrenia, or completely irrelevant to his presentation. Either way, it is neither specific nor diagnostic.
Shyness (option E) is not uncommon among children and teenagers and could be normal behavior. Shyness in history has no diagnostic significance or importance.
TOPIC REVIEW
According to DSM-V, diagnostic criteria for schizophrenia are as follows:
At least two of the following symptoms of which at least one is the first three:
1. Delusions
2. Hallucinations (almost always auditor)
3. Disorganized speech (e.g., frequent derailment or incoherence)
4. Grossly disorganized or catatonic behavior
5. Negative symptoms (i.e., flattened affect, alogia, or avolition)
Active symptoms must persist for ≥ 1 month (or less if successfully treated) while the continuous disturbance for ≥ 6 months.
Symptoms must cause social, occupational, or personal function impairment lasting ≥ 6 months.
Other possible causes for the symptoms are excluded.
Terminology
Childhood-onset schizophrenia – Childhood-onset (or very early-onset) schizophrenia starts prior to the age of 13 years.
Early-onset schizophrenia – Early-onset schizophrenia starts prior to age 18.
Adult-onset schizophrenia – Adult-onset schizophrenia starts at or after age 18.
A 17-year-old girl is diagnosed with major depression associated with psychotic features. She is planned to be started on selective serotonin reuptake inhibitors (SSRIs). For how long, should the treatment be continued?
A. Six months.
B. One year.
C. Five years.
D. Life-long.
E. Two years.
Correct Answer Is C.
All patients with major depression require continuation of treatment for up to 6 to 12 months;
however, the course of therapy should be extended in the following situations:
* Two depressive episodes within 5 years
* Three prior episodes
* Severe psychotic depression
* Serious suicidal attempt
If a patient is started on long-term treatment, this should probably be continued for at least 3 to 5
years, after which time the need for further management should be reviewed. Some patients may
even need life-long treatment.
A 27-year-old man is found to have major depression after he has psychiatric evaluation following an attempted suicide. He is started on sertraline. For how long he should receive the medication?
A. Six months.
B. One year.
C. Two years.
D. Five years.
E. Life-long.
Correct Answer Is D.
For most patients with depression the medical treatment should be continued for at least 6 months to ideally 12 months.
In the following situations, however, the duration of therapy should be extended to 3 to 5 years:
* Two episodes of major depression in 5 years
* Three previous episodes of major depression
* Depression with psychotic features
* Depression with a serious suicidal attempt
After 3-5 years, the patient should be reassessed for the need for further management. Some patient might need lifelong antidepressant therapy.
As this patient has had a suicidal attempt, he should receive antidepressant therapy for at least 3-5 years.
Which one of the following does not increase the risk of suicide?
A. Alcohol abuse.
B. Poor social support.
C. Inquiring the patient about suicidal ideation.
D. Conduct disorder.
E. Schizophrenia.
Correct Answer Is C.
A variety of factors are associated with an increased risk of suicide:
* Psychiatric disorders - Psychiatric illness is a strong predictor of suicide. More than 90% of patients who attempt suicide have a major psychiatric disorder, and 95% of patients who successfully commit suicide have a psychiatric diagnosis. Patients with psychiatric diagnoses kill themselves at rates 3 to 12 times higher than other patients.
* Hopelessness and impulsivity - Across psychiatric disorders, hopelessness is strongly associated with suicide. Hopelessness can persist even when other symptoms of depression have remitted. Impulsivity, particularly among adolescents and young adults, is also associated with acting on suicidal thoughts.
* History of previous suicide attempts or threats - The strongest single factor predictive of suicide is prior history of attempted suicide. Patients with a previous history of suicide attempts are 5 to 6 times more likely to make another attempt; furthermore, up to 50% of successful victims have made a prior attempt. One of every 100 suicide attempt survivors will die by suicide within one year of the previous attempt which amounts to 100 times that of the general population.
Age, sex, and race - The risk of suicide increases with increasing age; however young adults attempt suicide more often than older adults. Females attempt suicide 4 times more frequently than males, but males are successful 3 times more often. These age and sex differences appear to be primarily related to the lethality of the method chosen (e.g. firearms, hanging, jumping, etc.) rather than a difference in completion rates for the same method. Elderly white men, aged 85 years and older, and young black males have the highest suicide rate.
* Marital status - based on marital status the suicide risk in descending order is increased in:
1. Those who never married
2. Widowed, separated, or divorced
3. Married without children
4. Married with children
* Occupation - Unemployed and unskilled individuals are at higher risk for suicide than those who are
employed and skilled.
* Health - Suicide risk increases with physical illness such as chronic pain, recent surgery, and chronic or terminal disease.
* Adverse childhood experiences - Childhood abuse and other adverse childhood experiences appear to increase the risk of suicide in adults. Conduct disorders in children is shown to be associated with an increased risk of suicide
* Family history and genetics - The risk of suicide increases in patients with a family history of suicide. A first-degree relative who committed suicide increases the risk six-fold.
* Antidepressants - Antidepressants can have potential association with suicide.
* Other - The risk of suicide increases in following situations:
* Accessibility to weapons
* Sociopolitical, cultural, and economic forces
* Violence and political coercion
* Economic downturns
* Living in rural areas
* Being lesbian, gay, or bisexual
* Lower intelligence
Of the given options, inquiring the patient about suicidal ideation is not associated with an increased risk of attempting suicide. In fact, asking the patient directly about suicidal ideation is an essential part in assessment of suicide risk.
Which one of the following will not increase the risk of depression?
A. High socioeconomic status.
B. Elderly male with cognitive decline.
C. Unemployment.
D. Family history of depression.
E. Substance misuse.
Correct Answer Is A.
The following are the condition associated with increased risk of depression:
* Family history of depression
* Chronic illness
* Co-occurring mental conditions such as anxiety, personality disorders, etc
* Physical illness, physical or intellectual disability
* Low self-esteem, distorted body image, social incompetence
* Entering puberty and schooling, transition into workforce and independent living (adolescents)
* Language problems, generational culture clashes, cultural nonrecognition of mental health problems, stresses from living between two cultures
* Uncertainty, fear of rejection by family and friends, desire to ‘fit in’ with perceived societal expectations, being bullied, being subjected to homophobic abuse (more in adolescents)
* Domestic violence, poverty, family discord, sexual or physical abuse
* Bereavement, separation from loved ones, divorce, trauma
* Smoking, alcohol, drug use, internet use affecting sleep
* Marginalisation, homelessness, refugee status, fostering, unemployment
The high socioeconomic status is a protective factor against depression, not a risk factor.
A 24-year-old woman presents with history of low mood, psychomotor retardation, decreased appetite and decreased sleep for the past 2 weeks. She has family history of bipolar disorder.
Which one of the following is less common in bipolar depression than unipolar depression?
A. Psychomotor retardation.
B. Hypersomnia.
C. Decreased appetite.
D. Positive family history of bipolar disorder.
E. Delusions and hallucinations.
Correct Answer Is C.
The following features are more commonly seen in bipolar depression than unipolar depression:
* Psychomotor retardation
* Increased appetite (hyperphagia)
* Increased sleep (hypersomnia)
* Early onset of first depression before 25 years of age
* Delusions and hallucinations
* Positive family history of bipolar disorder
It is very important to identify these features in patients with possible diagnosis of bipolar disorder,
who initially present with depression. This patient has positive family history of bipolar disorder, psycho-motor retardation and age of onset of depression below 25 years, all favoring bipolar depression.
NOTE - Psychomotor retardation is seen in depressive phase of bipolar disorder. It is not a feature of mania associated with bipolar disorder. Decreased sleep and decreased appetite are not common features of bipolar depression and are seen in major depression more frequently.
Which of the following does not increase the risk of postaprtum depression?
A. Adverse life events.
B. Lack of social support.
C. Past history of depression.
D. Emergency cesarean section.
E. Elective cesarean section.
Correct Answer Is E.
Postpartum depression is most commonly seen during the first 1-8 weeks after delivery. The risk factors for development of postpartum depression (and postpartum anxiety disorders) include:
* Psychological
* Antenatal anxiety, depression or mood swings
* Previous history of anxiety, depression (option C), or mood swings, especially if occurred perinatally
* Family history of anxiety, depression or alcohol abuse, especially in first degree relatives
* Severe baby blues
* Personal characteristics like guilt-prone, perfectionistic, feeling unable to achieve, low selfesteem
* EPDS (Edinburgh postnatal depression) score ≥ 12
* Social Lack of emotional and practical support from partner and/or others (option B)
* Domestic violence, history of trauma or abuse (including childhood sexual assault)
* Many stressful life events recently (option A)
* Low socioeconomic status, unemployment
* Unplanned or unwanted pregnancy
* Expecting first child or has many children already
* Child care stress
* Biological / medical
* Recent cessation of psychotropic medications
* Medical history of serious pregnancy or birth complications (including emergency cesarean section (option D), neonatal loss, poor physical health, chronic pain or disability, or premenstrual syndrome
* Perinatal sleep deprivation
* Neonatal medical problems
Elective cesarean section does not increase the risk of postnatal depression.
A 76-year-old man is brought to your clinic by his son because of progressive decline in memory for the past 18 months, as well as weakness of his upper and lower limbs and deterioration of motor function. On examination, right hemiparesis is evident. Which one of the following could be the most likely diagnosis?
A. Alzheimer’s disease.
B. Lewy body dementia.
C. Multi-infarct dementia.
D. Parkinson’s disease.
E. Pick’s disease.
Correct Answer Is C.
The clinical picture is suggestive of multi-infarct dementia as the most likely cause. Multi-infarct
dementia is the second most common cause of dementia after Alzheimer disease (10% of all cases with dementia).
Alzheimer disease (option A), Lewy body dementia (option B) and Pick’s disease (frontotemporal dementia) (option E) are not associated with focal motor dysfunction. Parkinson’s disease (option D) as a cause of dementia is associated with motor dysfunction in the form of tremors and extrapyramidal symptoms. Features include gate disturbances, tremor, rigidity and micrographia.
Hemiparesis is not a feature.
TOPIC REVIEW
Diseases associated with dementia:
Alzheimer disease - Alzheimer disease is the most common cause of dementia. Typically, patients
with Alzheimer disease present with problems in memory and visuospatial abilities that occur early in the course of the disease. Despite severe memory impairment, social grace remains intact until very late in the course of the disease when hallucinations and personality changes develop.
Alzheimer disease is not associated with motor or sensory dysfunction at least not very late in the
course of the disease.
Lewy body dementia - Lewy body dementia is characterized by fluctuating level of consciousness,
social disinhibition and Parkinsonism. Dementia often follows later. Lewy body dementia can be confused with delirium.
Vascular dementia - Vascular dementia is divided into multi-infarct dementia, which typically has a stepwise progression associated with frequent discrete cerebrovascular events, and Binswanger
disease, involving the subcortical white matter, that presents with a slowly progressive course.
Normal pressure hydrocephalus - It presents with prominent gait abnormalities early in the course
of the disease that usually precedes the onset of memory impairment. Urinary incontinence is another distinguishing feature.
Pick’s disease (frontotemporal dementia) - Patients with Pick’s disease present with personality
changes early in the course of the disease, with relative sparing of visuospatial function. Social,
interpersonal, and emotional abnormalities precede memory impairment. The condition is first noted by the family because the patient does not have insight into their problem.
Parkinson’s disease - Dementia secondary to Parkinson disease is associated with typical features
of the disease such as gate disturbances, rigidity, tremors, micrographia, etc. Recurrent visual hallucinations can be a feature that usually develop later in the course of the disease.
Creutzfeldt – Jacob disease (CJD) - Dementia of CJD develops in shorter time (weeks to months)
and has a course more aggressive than Alzheimer’s disease. Myoclonus is a distinguishing feature.
Diagnosis of CJD is by rapidly progressive dementia, myoclonus and the presence of 14-3-3 protein
in the CSF.
Accompanied by his wife, a 63-year-old man presents to your practice for consultation. She is concerned about her husband because he has been recently behaving childish and bizarre. Last week he was dismissed from his job as a manager in a local restaurant, because of treating rude to customers and shouting at his colleagues. He does not shave, bathe or change his clothes as he did before and is disheveled and unkempt all the time. She denies any falls, gait abnormalities, or hallucinations in her husband. His memory is not significantly affected.
Which one of the following could be the most likely diagnosis?
A. Depression.
B. Alzheimer disease.
C. Lewy body dementia.
D. Frontotemporal dementia.
E. Schizophrenia.
Correct Answer Is D.
Cognitive and behavioral changes in aged people are frequently faced in general practice, with dementia and delirium being the most common underlying etiologies.
Cognitive function is measured by various mental functions, including memory, concentration,
praxis, language, executive functions, and visuospatial skills. Dementia refers to memory loss with
impairment of any other cognitive function that can interfere with social or occupational functioning.
A myriad of causes have been identified for dementia. These causes can be reversible or irreversible.
The most common reversible causes of dementia include:
* Hypothyroidism
* Vitamin B12 deficiency
* Hepatic or uremic encephalopathy
* Vasculitides affecting CNS
* Space occupying brain lesions i.e. abscess/tumors either primary or metastatic
* Medications – anticholinergics in particular
* Normal pressure hydrocephalus
* Central or obstructive sleep apnea
* Subdural hematoma
* Trauma
* Depression
Some of the most common irreversible causes of dementia are:
* Alzheimer disease (60-80% of cases )
* Vascular dementia including multi-infarct dementia and Binswanger disease
* Lewy body dementia
* Frontotemporal degeneration (dementia) including Pick disease
* Multifocal leukoencephalopathy
The case scenario describes a patient with social inappropriateness as the most concerning presenting symptom without memory being significantly involved. Of the options, the most consistent one with such scenario is frontotemporal dementia (Pick disease). In this disease, social disgrace is the earliest symptom with memory impairment and forgetfulness following later.
Frontotemporal dementia is characterized by focal degeneration of the frontal and/or temporal lobes. The typical age of onset is in the late 50s or early 60s, and the primary initial clinical manifestations are changes in personality and social behavior or language, progressing over time to a more global dementia. Other features include impaired initiation and planning, disinhibited behavior and social disgrace and mild abnormalities on cognitive testing. Apathy and memory deficits develop later in the course of the disease. A subset of patients may also exhibit symptoms of extrapyramidal or motor neuron involvement at some point in the disease process.
(Option A) Patients with depression may present with pseudodementia which is different from dementia in some aspects. It is less common for patients with pseudodementia to have disinhibition or social disgrace. The history of disturbances in pseudodementia is often short and abrupt onset, while dementia is more insidious. On cognitive testing, people with pseudodementia often answer that they do not know the answer to a question, and their attention and concentration are intact and they may appear upset or distressed. Those with true dementia will often give wrong answers, have poor attention and concentration, and appear indifferent or unconcerned.
(Option B) In Alzheimer disease, forgetfulness is usually the presenting symptom. It is very unlikely
for a patient with Alzheimer disease to present with disinhibition and social inappropriateness early in the course of the disease.
(Option C) although misbehavior and disinhibition is a common early feature in patients with Lewy body dementia, the absence of other manifestations such as fluctuating cognition, hallucinations, extrapyramidal deficits (Parkinsonism) and repeat falls makes this diagnosis less likely.
(Option E) Psychotic features such as hallucinations and delusion are a significant diagnostic component in schizophrenia that is absent here. Moreover, development of schizophrenia at this
age is unusual.
Which one of the following diseases of the central nervous system is caused by infectious proteins?
A. Alzheimer’s disease.
B. Creutzfeldt- Jakob disease.
C. Parkinson’s disease.
D. Pick’s disease.
E. Guillain-Barre syndrome.
Correct Answer Is B
Creutzfeldt-Jakob disease is caused by an infectious protein particle called prion. The disease is often contracted at about mid-seventies and presents with dementia and myoclonus. It has a progressive course and death follows in one year.
A 36-year-old man is brought to the emergency department by his relatives due to what they call an outburst. When you step in the examining room, you realise that he is very agitated and is yelling at the staff angrily and threatening to kill anyone who touches him. You are informed by the staff that he has the past history of schizophrenia and has been aggressive and violent at previous presentations. Which one of the following would be the next best step in management?
A. Tell the hospital security guards to hold him until he is calm.
B. Alert the hospital security to intervene and sedate him with haloperidol or benzodiazepine.
C. Call hospital security and request the patient to behave properly.
D. Ask the hospital security to escort the patient out of the hospital.
E. Hand him over to the police.
Correct Answer Is B.
Angry and agitated patients usually present a challenge to the health care providers. The best approach is to calm down the patient in tactful professional manners and ideally without use of sedatives; however when the patient is feared to pose harm to him/her or medical staff, hospital security is required to be called and the patient sedated.
Since this patient is high-risk for violence due to his psychiatric condition and past history of violence, calling the hospital security and sedating him would be the next best step in management.
In all Australian emergency departments, the’ zero tolerance policy to violence’ is adopted. This policy mandates sedation on the slightest concern of harm from the patients either to themselves or the staff. The medications of choice for this purpose include haloperidol (preferred) and midazolam. Midazolam carries the risk of respiratory depression and hypotension and should be used with great caution.
Benzodiazepines are first line when agitation is likely to have caused by alcohol intoxication or withdrawal. Patients in need of medical attention should be treated, not handed over to the Police, or escorted out.
During assessment of a 32-year-old woman, she says ‘Oh well, my food! My food is cheese, cheese is in the air, air is blue, and I came by bus’. Which one of the following can be the most likely condition she is suffering from?
A. Schizophrenia.
B. Depression.
C. Dissociation.
D. Psychosis.
E. Adjustment disorder.
Correct Answer Is D.
The scenario describes a typical example of ‘tangentiality’. Tangentiality occurs when one idea
connects to the next with one word or phrase, but the thoughts become confusing because they go off on a tangent and end in a different subject. In the above example the word ‘food’ ends in another comment about food unrelated to the previous sentence. The word ‘food’ is the only connection. Loosening of association (derailment), on the other hand, happens when one idea does not connect to the next at all.
Following are sentences told by patients in real situations. The first two are examples of derailment
(loosening of association), while the third is ‘tangentiality’:
- The next day when I’d be going out you know, I took control. Like uh, I put bleach on my hair …
- The traffic is rumbling along the main road. They are going to the north. Why do girls always play pantomime heroes?
- I think someone has infiltrated my copies of the cases. We’ve got to case the joint. I don’t believe in joints, but they do hold your body together.
Derailment, loosening of association, poverty of content of speech, and thought blocking are
examples of disorganized thought content as characteristic features of psychosis and psychotic
disorders. A brief psychotic episode, schizophreniform disorder, schizoaffective disorder and schizophrenia can have tangentiality/derailment as a presentation. Psychosis is the option encompassing all these disorders and the correct answer for this question.
An alcoholic man is brought to your clinic because of alcohol intoxication. He is successfully resuscitated. Which one of the following conditions in the history will direct you towards alcohol dependence in this patient?
A. Any compulsory alcohol drinking first thing in the morning.
B. He drinks when he is anxious.
C. He drinks socially.
D. He drinks when he is driving.
E. He drinks more than 4 standard drinks in one single session.
Correct Answer Is A.
There are screening tools to assess the likelihood of alcohol dependence in primary care setting.
CAGE questionnaire and AUDIT (Alcohol Use Disorders Identification Test) questionnaires are the
two most commonly use tools for this purpose.
The CAGE test consists of 4 questions. The letters of the acronym are the initial letters of key
words in questions:
1. Have you ever felt the need to CUT down on your drinking?
2. Have you ever felt ANNOYED by others asking you about your drinking?
3. Do you feel GUILTY about your drinking?
4. Do you ever have an EYE-OPENER in t the morning?
A positive response to any item on the CAGE questionnaire is a pointer towards alcohol
dependence and warrants a detailed assessment.
AUDIT questionnaire consists of 10 questions and a maximum score of 40. A score of 8 or more out of the total score of 40 suggests alcohol dependence. AUDIT questionnaire has minimal false positive or negative results.
Of the given options, compulsory morning drinking (eye opener) is suggestive of alcohol dependence.
Other options may or may not be seen in alcohol dependence and are not useful in determining the
possibility of alcohol dependence.
Jane, 32 years, gave birth to her child 3 days ago and has been in the ward since. On the second day, she went to the nurses and told them that she is very worried and afraid about her child and needs to keep her safe in the room. Last night she was seen walking around and moving objects whole night. She took an insecticide and sprayed on the floor to get rid of imaginary bugs. On examination, she is agitated with a heart rate of 110 bpm, respiratory rate of 22 breaths per minute and temperature of 37.5°C. Which one of the following is most important initial step in management?
A. Blood culture.
B. Full blood count (FBC).
C. Urine drug screen.
D. CT scan of the head.
E. Urine analysis (UA).
Correct Answer Is C.
The scenario is consistent with diagnosis of an acute psychotic episode in early postpartum period. A few explanations should be considered and investigated. The most important ones include:
* Substance-induced psychotic disorder (substance intoxication/withdrawal)
* Brief psychotic disorder
* Postpartum psychosis
Considering the time of the presentation (shortly after delivery) and other physical finding, drug intoxication as a likely cause should be excluded first. For this purpose, a urine drug screen test is the most appropriate measure.
A rare yet important diagnosis to consider is postpartum psychosis. The condition most commonly
presents within 2 weeks of childbirth. Hallucinations and delusions are usually present, often with thought disorganization and/or bizarre behavior.
Although one should have postpartum psychosis as one possible diagnosis, more prevalent etiologies for an acute psychotic episode should be consider and excluded first, especially with the very early onset of symptoms which seems somewhat unusual (but not impossible) for postpartum psychosis.
(Options A, B and E) With a temperature of 37.5°C, it is very unlikely that an infectious process is
the cause of this presentation; hence, blood culture, FBC, and UA are not appropriate steps; at least not as the most important initial approach.
(Option D) CT scan of the head would have been indicated as a part of initial management if there was a pointer towards an intracranial pathology.
Alcohol-related dementia acounts for what percent of all cases of dementia in Australia?
A. 10%.
B. 5%.
C. 90%.
D. 80%.
E. 35%.
Correct Answer Is B.
Incidence of alcohol-related dementia in Australia is about 5% (5.4% according to a study by Panegyres and Frencham (2000)) of total demented population. Vascular dementia accounts for 10% and Alzheimer for 80-85% of all cases of dementia in Australia.
A 23-year-old woman is referred to you for psychiatric assessment. When you ask her what she
thinks to be her problem, she answers: “Oh, isn’t it a long story? The train always runs on rails; the birds keep flying up the hills and I have to remember to count my chickens before they hatch.” At the end she does not come back to answer the question you asked. Which one of the following is the most likely thought disorder she has?
A. Derailment.
B. Flight of ideas.
C. Pressured speech.
D. Circumstantiality.
E. Word salad.
Correct Answer Is A.
Derailment (Loosening of Associations):
- Definition: No connection between sentences; one unrelated sentence follows another.
- Example: “I went to the store. The sky is blue.”
Tangentiality:
- Definition: Milder form of derailment with a linking hint between sentences but no return to the initial topic.
- Example: “I think someone has infiltrated my copies of the CASEs. We’ve got to CASE the JOINT. I don’t believe in JOINTs, but they do hold your body together.”
Flight of Ideas:
- Definition: Over-productive speech with rapid shifts between topics, often with a connecting hint.
- Example: Rapid speech where the patient’s thoughts seem to race.
Pressured Speech:
- Definition: Rapid and urgent speech that’s hard to interrupt.
- Example: Talking non-stop and too fast for others to understand.
Circumstantiality:
- Definition: Drifting from the topic but eventually returning to the original point.
- Example: Long-winded story that eventually answers the question.
Word Salad:
- Definition: Jumbled words without sensible meaning.
- Example: “Blue sky jump river laugh.”
A 30-year-old patient says: “I went to the store. The sky is blue.”
-
What is the speech pattern?
- A) Tangentiality
- B) Flight of Ideas
- C) Pressured Speech
- D) Derailment
- E) Word Salad
- Why not A): Tangentiality has a linking hint between sentences.
- Why not B): Flight of Ideas involves rapid, organized shifts between topics.
- Why not C): Pressured Speech is rapid and urgent.
- Why not E): Word Salad is jumbled words without meaning.
A patient says: “I love cats. Cats chase mice. Mice are found in houses.”
-
What is the speech pattern?
- A) Derailment
- B) Tangentiality
- C) Circumstantiality
- D) Pressured Speech
- E) Word Salad
- Why not A): Derailment has no connection between sentences.
- Why not C): Circumstantiality returns to the starting point.
- Why not D): Pressured Speech is urgent and rapid.
- Why not E): Word Salad lacks sensible meaning.
A patient rapidly shifts from talking about their job, to their vacation, to their favorite food.
-
What is the speech pattern?
- A) Derailment
- B) Tangentiality
- C) Flight of Ideas
- D) Pressured Speech
- E) Word Salad
- Why not A): Derailment has unrelated sentences.
- Why not B): Tangentiality has linking hints.
- Why not D): Pressured Speech is rapid but not necessarily shifting topics.
- Why not E): Word Salad lacks coherent meaning.
A patient talks non-stop about various topics at a rapid pace and cannot be interrupted.
-
What is the speech pattern?
- A) Derailment
- B) Tangentiality
- C) Flight of Ideas
- D) Pressured Speech
- E) Word Salad
- Why not A): Derailment has unrelated sentences.
- Why not B): Tangentiality has linking hints but is less urgent.
- Why not C): Flight of Ideas involves rapid topic changes but is not necessarily urgent.
- Why not E): Word Salad lacks coherent meaning.
A patient takes a long, roundabout way to answer a simple question but eventually answers it.
-
What is the speech pattern?
- A) Derailment
- B) Tangentiality
- C) Flight of Ideas
- D) Pressured Speech
- E) Circumstantiality
- Why not A): Derailment never returns to the initial topic.
- Why not B): Tangentiality has linking hints and doesn’t return to the initial topic.
- Why not C): Flight of Ideas is rapid and coherent but involves frequent topic changes.
- Why not D): Pressured Speech is rapid and difficult to interrupt.
The speech pattern given in the question is characteristic of derailment, also known as “loosening
of associations”, in which there is no connection between one topic and the next. One sentence is spoken and then another sentence unrelated to the previous one follows.
Tangentiality is a milder form of derailment in which there is a linking hint between the two consecutive sentences. An example is: I think someone has infiltrated my copies of the CASEs. We’ve got to CASE the JOINT. I don’t believe in JOINTs, but they do hold your body together.
Words in upper case are the links between each sentence and the previous one. These connecting hints are characteristic of tangentiality. In tangentiality (like derailment) there is no returning to the initial topic.
(Option B) Flight of ideas is characterized by over-productive speech with rapid shifting from one topic to another. There is often a hint in the previous topic leading to the next one. In the flight of ideas, there is a subjective feeling that the thoughts are racing. In flight of ideas, the topic spoken by the patient is organized but over-productive and in excess details. In other words, the general concept of the current topic is adequately understood, but a hint in one part leads the patient to another topic. The flight of idea is a matter of switching between coherent ideas.
(Option C) Pressure of speech (pressured speech) is a tendency to speak rapidly and excitedly, as if motivated by an urgency not apparent to the listener. The speech is difficult to interrupt. It may be too fast, or too tangential for the listener to understand.
(Option D) Circumstantiality occurs when the patient drifts from one topic to the other, but eventually returns to the starting point. In other words, if a question is imagined as a destination, there are many detours, but the destination is reached at the end. Unlike circumstantiality, patients with derailment (loosening of association) never come back to the topic they started off with.
Tangentiality or tangential speech is a milder form of derailment in which there is a hint linking two consecutive topics.
(Option E) In word salad, the patient throws words together without any sensible and intelligible
meaning.
A 47-year-old homeless man is brought to the Emergency Department by the police after he was
found agitated and fearful in the streets while carrying a bag full of rubbish. He says to you, as the
treating doctor, that he was caught by special military forces because he did not returned the DVDs to a shop that belongs to the Jewish brotherhood. He also mentions that he has been followed by demons because he has not done his prayers. He does not seem to be drunk but looks exhausted and agitated. Which one of the following options is the most appropriate next step in management of this patient?
A. Admit him to psychiatric unit for evaluation and treatment.
B. Ask the police about him.
C. Arrange urgent parenteral antipsychotics.
D. Arrange admission to general ward for assessment of his medical status.
E. Perform a urinary toxicology screen.
Correct Answer Is B.
Here’s a simplified explanation:
When evaluating an agitated patient in a psychiatric setting, the following steps are important:
- De-escalation: The first step is to calm the patient. This can be done through verbal techniques (talking to the patient calmly) or, if necessary, using medication to help them relax.
- Observation: Before talking directly to the patient, observe them visually. Pay close attention to how they act and communicate, both verbally and nonverbally, during the de-escalation process.
- Gathering Information: Collecting information from others can be very helpful. While the patient is being calmed, another team member can talk to family members, paramedics, or police officers who might have useful information. Reviewing the patient’s medical records, especially electronic ones, can also provide important details about their history, previous diagnoses, and medications. Among these options, asking the police is often a priority because they may have critical information about the patient’s behavior.
- Assessing for Delirium: After confirming that the patient does not have an acute medical problem, it’s important to check for delirium, a condition that can cause confusion, difficulty focusing, and other cognitive issues. Delirium is sometimes mistaken for psychosis, so it’s essential to carefully observe the patient’s behavior for signs like confusion, trouble focusing, or responding to things that aren’t there (hallucinations). If these symptoms are present, especially if there’s a history of drug use, the likely diagnosis is delirium.
- Suicide or Self-Harm Risk: It’s crucial to assess if the patient is at risk of harming themselves or others and manage this risk accordingly.
- Check for Substance Issues: Consider whether the patient might be intoxicated or experiencing withdrawal from substances, and investigate if needed.
- Medical and Psychiatric Assessment: Evaluate the patient’s overall medical condition. If an underlying psychiatric condition is suspected, a more detailed psychiatric assessment should be conducted.
- Medication: If verbal calming techniques do not work, antipsychotic medications may be used to help manage the patient’s agitation.
Psychiatric evaluation of the agitated patient includes:
De-escalation (verbal de-escalation; medical deescalation [e.g. sedation] if necessary)
Visual observation of the patient before direct patient interview and paying careful attention to the patient’s verbal and nonverbal interaction with the examiner during de-escalation.
Collecting collateral information - Collecting collateral information can be very helpful. While de-escalation is in process, another team member can obtain verbal reports from family, paramedics, or police officers or review written material that may accompany the patient.
Medical records are also an important source of information, and electronic records, if available, can be readily accessed to determine previous diagnoses and medications. These sources of information can be invaluable in determining the cause of agitation. Of the given options, asking the police about him has the highest priority as it can provide additional information for further management.
Once it is determined that the patient does not have an acute medical problem, there are several important questions, the answers to which will guide the next step in management of the patient. The first question is whether the patient has a delirium. It is not uncommon for a patient to go through initial screening and have a diagnosis of delirium overlooked. The patient may be mistakenly diagnosed as being psychotic, or the signs and symptoms of delirium may be subtle and easily overlooked. In delirium, the patient has an altered level of awareness and problems directing, focusing, sustaining, or shifting attention. The examiner must pay close attention to how the patient interacts during the encounter and recognize these often subtle signs. Does the patient seem confused and unable to focus? Are there perseverative behaviors? Does the patient appear to be responding to visual hallucinations? Are there signs of language impairment, problems naming, or other cognitive deficits? If agitation is associated with any of these findings, especially in the
setting of drug or medication use or medical illness, the presumptive diagnosis is delirium.
Risk of suicide or self-harm should be assessed and if presents managed accordingly. The possibility of substance intoxication/withdrawal should be borne in mind and investigated if indicated.
Other measures include assessment of general medical condition, followed by a detailed formal
psychiatric assessment if an underlying psychiatric condition is suspected. Antipsychotics may be used for treatment of an agitated patient if verbal de-escalation fails.
A 35-year-old man presents to your practice with complaint of impotence for the past few months. He smokes 20 cigarettes and regularly drinks 3-4 glasses of wine a day. He is marrying his fiancee in one month. Which one of the following would be the most appropriate management option for him at this stage?
A. Cessation of smoking.
B. Cessation of alcohol drinking.
C. Referral for psychotherapy.
D. Prescribe sildenafil.
E. Prescribe clomipramine.
Correct Answer Is D.
Unless contraindicated, oral phosphodiesterase type 5 (PDE5) inhibitors such as sildenafil, vardenafil, tadalafil and avanafil are first-line therapy for erectile dysfunction, regardless of the etiology. In patients refractory to oral PDE5 inhibitors, one of these agents can be combined with an injection of the prostaglandin PGE1.
Although not proven, it is likely that erectile dysfunction can be prevented by good general health,
paying particular attention to body weight, exercise, and cigarette smoking and alcohol consumption. These measures should be considered but are not first-line treatment plan.
TOPIC REVIEW
Impotence is defined as male erectile dysfunction (ED), that is, inability to achieve or maintain an erection sufficient for satisfactory sexual performance.
ED usually has a multifactorial etiology. Organic, physiologic, endocrine, and psychogenic factors are involved in the ability to obtain and maintain erections. In general, ED is divided into 2 broad categories, organic and psychogenic. Although psychologic factors have been mostly implicated
as the contributing factor, pure psychogenic ED is in fact uncommon; however, many men with organic etiologies may also have an associated psychogenic component.
Conditions that may be associated with ED include diabetes, hypertension, and coronary artery disease as well as neurologic disorders, endocrinopathies, benign prostatic hyperplasia, sleep apnea, COPD, and depression. In fact, almost any disease may affect erectile function by altering the nervous, vascular, or hormonal systems. Various diseases may produce changes in the smooth muscle tissue of the corpora cavernosa or influence the patient’s psychological mood and behavior.
Conditions associated with reduced nerve and endothelium function (e.g. aging, hypertension, smoking, hypercholesterolemia, and diabetes) alter the balance between contraction and relaxation factors. These conditions cause circulatory and structural changes in penile tissues, resulting in
arterial insufficiency and defective smooth muscle relaxation. In some patients, sexual dysfunction may be the presenting symptom of these disorders. Given the multiplicity of possible etiologic factors, it may be difficult to determine how much any given factor is contributing to the problem. A thorough evaluation is necessary for correct identification of the specific cause or causes in any given individual.
NOTE – hypertension has been found to have the highest association with ED. Benign prostatic hyperplasia (BPH) has also a strong association. The cause exact mechanism, however, is unclear.
The first step in the management of ED is a thorough history that includes the following:
* Sexual history
* Medical history
* Psychosocial history
A physical examination is necessary for every patient, emphasizing the genitourinary, vascular, and neurologic systems. A focused examination entails evaluation of the following:
* Blood pressure
* Peripheral pulses
* Sensation
* Status of the genitalia and prostate
* Size and texture of the testes
* Presence of the epididymis and vas deferens
* Abnormalities of the penis (e.g. hypospadias, peyronie plaques)
Management options for ED include the following:
* Sexual counseling, if no organic causes can be found for the dysfunction. Sexual counselling, however, can be used as an ongoing treatment along with other therapies
* Oral medications
* Injected, implanted, or topically applied medications
* External vacuum and constriction devices
* Surgery (penile prosthesis, vascular surgery)
A 17-year-old boy is brought to your office by his mother for evaluation. History revleas that he had
always been a good student until the age 15 when his grades began dropping. His mother has also noticed that he has stopped socializing with friends and begun spending more time alone in his room. He dropped the school earlier this year due to poor school performance. His mother reports that he has been trying to home-school him, but he does not seem interested in learning. Days go before he takes bath and he does not seem to care about his appearance. What concerns the mother most and has led her to bringing him to see a psychiatrist is the fact that for the past 2 months, she has overheard him having what sounds like conversations with someone imaginary in his room. Which one of the following could be the most likely diagnosis?
A. Major depressive disorder.
B. Substance abuse disorder.
C. Schizophrenia.
D. Brief psychotic disorder.
E. Schizophreniform disorder.
Correct Answer Is C.
The scenario, describes a condition of 2-year duration, with social withdrawal as the initiating event (dropping grades and poor school performance), refusing to home-learn, unkempt appearance, and more importantly conversations with an imaginary individual that is a strong pointer towards hallucinations. These features are characteristic of schizophrenia as the most likely diagnosis.
To establish the diagnosis of schizophrenia the following criteria should be met:
Two (or more) of the following symptoms:
* Hallucinations
* Delusions
* Disorganized speech
* Grossly disorganized or catatonic behavior
* Negative symptoms
AND
* The symptoms are severe enough to cause significant social or occupation disability
AND
* The symptoms have lasted for at least 6 months
AND
* Other psychotic conditions such as schizophreniform disorder, brief psychotic episode, etc. are excluded
AND
* Drugs or medical conditions as the cause of symptoms are excluded
AND
* The condition could not be solely attributed to pervasive developmental disorders such as
autistic disorders
This young man’s conversations with imaginary represents hallucinations. His lack of motivation,
isolation, and poor self-care are examples of negative symptoms for 2 years or more. He has been like this for more than 6 months now and there is marked social or occupational dysfunction as evidenced by his poor school performance. These fulfill the criteria for schizophrenia as the diagnosis.
(Option A) Major depressive disorder can present with psychotic symptoms occurring in the context of a major depressive episode. The patient’s history does not suggest major depression. He has had 2 years of negative symptoms followed by positive symptoms, suggesting schizophrenia rather than major depression.
(Option B) A medical or substance-related condition is always among the differential diagnoses in patients with psychotic features. There is no hint about a medical condition as the underlying cause in the scenario, and it is unlikely that substance abuse of a teenager who lives with his parents goes unnoticed for 2 years.
(Option D) Brief psychotic disorder can share many features with schizophrenia. The distinctive
feature; however, is the symptoms duration of less than one month with eventual return to premorbid functioning.
(Option E) Although schizophreniform disorder shares many features with schizophrenia, the symptoms last no longer than 6 months in schizophreniform disorder.
A 50-year-old man presents to your practice, significantly concerned about amnesia. He relates that he has been to a place for work for the first time with one of his colleagues. He thought that this is
his first time he had ever visited that place but his colleague was surprised and told him that he had visited this place before and wondered why he could not remember. His past medical history is significant for electroconvulsive therapy (ECT) sessions due to major depression with last session being 3 months ago. Which one of the following in the history best justifies his presenting complaint?
A. History of dementia in his cousin.
B. History of depression.
C. Previous ECT sessions.
D. De javu.
E. Relapse of his major depression.
Correct Answer Is C.
Of the given options, previous ECT sessions are the most appropriate and justifying explanation to this man’s current problem that is forgetting past events i.e. retrograde amnesia.
Electroconvulsive therapy (ECT) is a treatment option for several psychiatric conditions. Major depression associated with psychotic features, strong suicidal ideation, unresponsiveness to other therapies and previous favorable response to ECT are the most frequent indications.
Most patients report some adverse cognitive effects during and after a course of ECT. The incidence depends upon electrode placement, stimulus type and dose, anesthesia, and the patient’s pretreatment cognitive status. A systematic review found four studies in which the proportion of patients who reported any memory loss ranged from 51 -79%.
ECT can cause three types of cognitive impairment:
Acute confusion - the acute confusional state is the result of both the seizure and the anesthesia. It
typically resolves 10 to 30 minutes after the procedure.
Anterograde amnesia - anterograde amnesia is the decreased ability to retain newly acquired information. It occurs during a course of ECT and typically resolves within two weeks after completing the course.
Retrograde amnesia - retrograde amnesia is defined as forgetting recent memories and is the most anxiety-producing and frightening cognitive effect of ECT. The affected memories are for events
that occur during the course of ECT and a period of weeks to a few months prior to that. The deficits are greatest and most persistent for knowledge about public or world events compared with knowledge about the self (personal memory). Bilateral ECT causes more retrograde amnesia than right unilateral. Retrograde amnesia recovers more slowly than anterograde amnesia. Some of the lost memories of events prior to the course of ECT may be expected to return, while others may not.
(Option A) Although dementia starts by disorientation, typically first to places, this man is young for dementia, and while dementia in a first-degree relative can be risk factor, a demented cousin does not appear to be of importance.
(Options B and E) Neither the history of depression in itself, nor a relapse of depressive episode justifies dementia.
(Option D) In Déjà vu is the sense familiarity with unfamiliar things. For example the patient feels that he/she has been in place before while he/she has not. This is not the case here.
Which one of the following is not considered a basic investigation for patients with anorexia nervosa?
A. Electrolytes.
B. Urinalysis.
C. Liver function tests.
D. DXA scan.
E. Electrocardiogram (ECG).
Correct Answer Is D.
Anorexia nervosa is characterized by failure to maintain a normal body weight, fear of weight gain and preoccupation with a distorted body image and unrealistic self-evaluation as overweight. There are two types: (1) restrictive and (2) binge eating/purging.
In restrictive type, the patient strictly reduces food intake and maintains diets of low-calorie foods. In purging type, weight loss is achieved through vomiting, laxatives, diuretics, or enemas.
Clinical manifestations of anorexia nervosa are mostly related to prolonged starvation and malnutrition, and include:
* Hypothermia
* Acrocyanosis
* Resting bradycardia (resting heart rate often 40-49 beats per minute)
* Hypotension
* Orthostatic lowered blood pressure or pulse
* Loss of muscle mass
* Low blood glucose (impaired insulin clearance)
* Low parathyroid hormone levels
* Elevated liver function tests
* Low white blood cell (WBC) count
Since eating disorders are clinical diagnoses, no definitive diagnostic tests are available for anorexia nervosa; however, with negative impacts of starvation on different multiple organ systems, the following basic tests should be performed for every patient with anorexia nervosa as baseline tests:
* Physical and mental status evaluation
* Full blood exam (FBE) and ESR
* Metabolic panel (Na, K, Bun, urea, Cl, BS, HCO3)
* Urinalysis for ketones and specific gravity
* Liver function tests
* Pregnancy test (in patients of childbearing age)
Both dehydration and increased fluid intake can occur in anorexia nervosa, leading to electrolytes imbalances with negative impacts on cardiac conduction system, central nervous system, etc.
Therefore, electrolyte assessment (option A) should be considered in all patients with anorexia
nervosa and bulimia nervosa.
Cardiac problems is the leading cause of death in patients with anorexia nervosa and an ECG (option E) should be obtained in all such patients as a very important basic assessment. In anorexic patients with BMI less than 14kg/m2, an echocardiography is mandated.
Urinalysis (option B) is another component of basic assessment. Specific gravity reflects hydration
status, and ketonuria indicates starvation.
Prolonged starvation has deleterious effects on liver function; hence, liver function tests (option C)
are indicated as basic investigations in all patients with anorexia nervosa.
Patients with anorexia nervosa and bulimia nervosa may have central hypogonadism and estrogen deficiency. Estrogen deficiency can result in osteopenia and osteoporosis. A Bones scan (DXA
scan) is indicated for all patients with anorexia nervosa of longer than 6 months duration; this test however is not among base line investigations. It can be considered later in the course of management.
Which one of the following features does not help to differentiate mania from hypomania?
A. Delusions.
B. Hallucinations.
C. Functional impairment.
D. Need for hospitalization.
E. Elevated mood.
Correct Answer Is E.
Features of mania versus hypomania is summarized in the following table: See table below.
Elevated mood is seen both in mania and hypomania and does not help to differentiate between these two.
Mania versus hypomania is the parameter by which bipolar disorder is classified into I and II.
Patients with mania are calssified as bipolar I disorder, whereas those with hypomania are labeled as having bipolar II disorder.
A 17-year-old girl has bizarre eating behavior noted by her mother. She indulges in episodes of binge eating at least once or twice a week. She spends several hours a day doing excessive workouts, and follows a restrict low-calorie diet because she believes she is fat. She also spends considerable amount of time in front of mirror checking her body. On examination, she has a BMI of 19 kg/m2. Which one of the following could be the most likely predisposing factor to this presentation?
A. Alcohol abuse.
B. Childhood sexual abuse.
C. Family history of obesity.
D. Family history of athleticism.
E. History of substance abuse.
Correct Answer Is B.
The case scenario describes a typical case of bulimia nervosa, indicated by binge eating, compensatory behavior to lose weight, and normal BMI that differentiates it from binging/purging type of anorexia nervosa.
There is no consensus on the causes of eating disorders. A combination of genetic, biologic, psychological, family, environmental, and social factors probably contribute to developing an eating
disorder.
Factors associated with the development of eating disorders include:
* Genetics – genetic factors are involved in the pathogenesis of eating disorders. As an example, concordance for either anorexia nervosa or bulimia nervosa is greater in monozygotic twins compared with dizygotic twins.
* Family distress – family characteristics associated with eating disorders may include high perceived parental expectations for achievement and appearance, families who communicate poorly, have members who are enmeshed with or estranged from each other, devalue the mother or maternal role, have marital tension, or have difficulty managing conflicts.
* Sexual abuse
* History of dieting
* Preoccupation with a thin body and social pressure about weight
* Athletic and artistic endeavors such as running, ballet, etc. that emphasizes leanness, and sports in which scoring is partly subjective (e.g., skating or gymnastics)
Of the given options, sexual abuse is the most important risk factor for development of eating disorders, as well as other psychiatric conditions such as substance abuse, depressive disorders, and anxiety disorders.
Sexual abuse can have many different effects on eating habits and body image of survivors. Sexual abuse violates the boundaries of the self so dramatically that inner sensations of hunger, fatigue,
or sexuality become difficult to identify. Victims of sexual abuse may turn to food to relieve a wide range of different states of tension unrelated to hunger. It is their confusion and uncertainty about their inner perceptions that leads them to focus on the food.
Many survivors of sexual abuse often work to become very fat or very thin in an attempt to render themselves unattractive. In this way, they try to de-sexualize themselves. Other survivors obsessively diet, starve, or purge to make their bodies “perfect.” A perfect body is their attempt to feel more powerful, invulnerable, and in control, so as not to re-experience the powerlessness they felt as children.
Several studies have found that childhood trauma such as childhood sexual abuse is a risk factor
for onset of bulimia nervosa as well as discontinuation of psychotherapy for bulimia nervosa; In addition, a history of childhood maltreatment in patients with bulimia nervosa is associated with comorbid depressive disorders, anxiety disorders, and borderline personality disorder.
In all eating disorders, there is an increased genetic heritability and frequency of a family history. A family history of ‘leanness’ or ‘thinness’ or ‘elite athleticism’ (option D) may be associated with anorexia nervosa, whereas bulimic eating disorders are associated with a personal or family history of obesity (option C). Although these risk factors are specific to anorexia nervosa and bulimia nervosa, they are less likely to trigger the eating disorder compared to sexual child abuse.
Substance abuse (option E) for weight reduction has been frequently observed among patients with anorexia nervosa and bulimia nervosa as a consequent of the disorder, but not as an underlying etiology.
Alcohol abuse (option A) does not seem to increase the risk of developing anorexia/bulimia nervosa.
Which one of the following is the most important risk factor for developing anorexia nervosa?
A. Alcohol abuse.
B. Childhood sexual abuse.
C. Family history of obesity.
D. Family history of athleticism.
E. History of substance abuse.
Correct Answer Is B
- Significance: Most important risk factor for bulimia nervosa, anorexia nervosa, and other eating disorders, as well as psychiatric conditions like substance abuse, depression, and anxiety.
- Effects: Victims may struggle to identify feelings of hunger, fatigue, or sexuality, turning to food to manage tension unrelated to hunger.
- Body Image: Some survivors try to become very thin or very fat to desexualize themselves or achieve a perfect body for control and empowerment.
-
Connection to Eating Disorders:
- Bulimia Nervosa: Strong association (47-50% prevalence).
- Anorexia Nervosa: Weaker association (23-27% prevalence).
- Anorexia Nervosa: Linked to family history of “leanness” or “thinness,” possibly elite athleticism.
- Bulimia Nervosa: Associated with personal or family history of obesity.
- Alcohol Abuse (Option A): Not a direct cause of anorexia/bulimia nervosa, though substance abuse can occur as a consequence.
- Substance Abuse (Option E): Often observed in patients with eating disorders but not the underlying cause.
- Family History: Increased genetic heritability in all eating disorders.
- Sexual Abuse: Primary risk factor for developing eating disorders and psychiatric conditions.
- Childhood Trauma: More strongly associated with bulimia nervosa than anorexia nervosa.
- Genetic Factors: Influence family history of body types, impacting risk for anorexia and bulimia nervosa.
Understanding these factors helps in recognizing and addressing the complex influences contributing to eating disorders.
Of the given options, sexual abuse is the most important risk factor for development of bulimia
nervosa, anorexia nervosa and other eating disorders, as well as other psychiatric conditions such
as substance abuse, depressive disorders, and anxiety disorders.
Sexual abuse can have many different effects on the eating habits and body image of survivors.
Sexual abuse violates the boundaries of the self so dramatically that inner sensations of hunger,
fatigue, or sexuality become difficult to identify. Victims of sexual abuse may turn to food to relieve
a wide range of different states of tension unrelated to hunger. It is their confusion and uncertainty
about their inner perceptions that leads them to focus on the food.
Many survivors of sexual abuse often work to become very fat or very thin in an attempt to render
themselves unattractive. In this way, they try to desexualize themselves. Other survivors
obsessively diet, starve, or purge to make their bodies “perfect.” A perfect body is their attempt to
feel more powerful, invulnerable, and in control, so as not to re-experience the powerlessness they
felt as children.
Several studies have found that childhood trauma such as childhood sexual abuse is a risk factor
for onset of bulimia nervosa as well as discontinuation of psychotherapy for bulimia nervosa.
Studies show the connection between childhood sexual abuse and anorexia nervosa is not as
strong as that observed in bulimia nervosa; however, child sexual abuse remains a very important
risk factor for all eating disorders including anorexia nervosa.
Two studies showed that a history of childhood sexual abuse was present in 47-50% of patients
with bulimia nervosa and a less strong connection between childhood sexual abuse and anorexia
nervosa when compared with that of bulimia nervosa (23-27%); In addition, a history of childhood
maltreatment in patients with bulimia nervosa is associated with comorbid depressive disorders,
anxiety disorders, and borderline personality disorder.
In all eating disorders there is an increased genetic heritability and frequency of a family history. A
family history of ‘leanness’ or ‘thinness’ or ‘elite athleticism’ (option D) may be associated with
anorexia nervosa, whereas bulimic eating disorders are associated with a personal or family history
of obesity (option C). Although these risk factors are specific to anorexia nervosa and bulimia
nervosa, they are less likely to be trigger the eating disorder compared to sexual child abuse.
Substance abuse (option E) for weight reduction has been frequently observed among patients
with anorexia nervosa and bulimia nervosa as a consequent of the disorder, but it is not the
underlying etiology.
Alcohol abuse (option A) does not seem to increase the risk of developing anorexia/bulimia
nervosa.
A 67-year-old woman comes to your GP clinic 6 weeks after her husband passed away due to
prostate cancer. She is sad and tearful and complains that she cannot sleep at night and is also
afraid of the dark. She mentions similar feelings 32 years ago after she had a still birth. Which one
of the following will help with prompt relief of her symptoms?
A. Venlafaxine.
B. Electroconvulsive therapy (ECT).
C. Olanzapine.
D. Temazepam.
E. Citalopram.
Correct Answer Is D
Given the symptoms and their duration, a normal grief reaction is the most likely explanation to this
woman’s problem.
Sadness, despair, tearfulness, decreased sleep, decreased appetite, and decreased interest in life
and the world are present as common findings in normal grief reaction. Visual and auditory
hallucinations of the deceased person are common and may lead the bereaved person to fear he or
she is ‘going crazy’. Guilt and shame are not common in normal grief reaction, yet possible.
Suicidality is not usually a concern
Patients with normal grief reaction return to normal social functioning within 2 months; the
symptoms, however, might last up to one year with waxing and waning.
Conventional support (e.g., family, social, religious, social) are usually sufficient to help the patient
over her grief. Although there is normally little place for pharmacotherapy, anxiolytics and
hypnotics can provide prompt relief where indicated due to excessive insomnia and anxiety. A
short course of benzodiazepines (< 7 days) can be used to help with the insomnia and anxiety.
The patient has no psychotic feature necessitating antipsychotics such as olanzapine (option C).
Antidepressants such as venlafaxine (option A) and citalopram (option E) might be indicated in
patients with complicated grief or grief associated with major depression, none of which is the
case here.
ECT (option B) might very infrequently be indicated in selected patients with complicated grief
reaction.
A 32-year-old woman presents to your practice with complaint of back pain, felt between her
shoulder blades. Her past medical history is remarkable for anorexia nervosa for 16 years. On
examination, she is severely emaciated, weighs 38 kg and has a body mass index (BMI) of 14
kg/m2
. Palpation of the interscapular region elicits tenderness over the vertebral column. Which
one of the following could be the most likely diagnosis?
A. Vertebral compression fracture.
B. Pneumonia.
C. Pulmonary embolism.
D. Pericardial effusion.
E. Metastatic bone lesion.
Correct Answer Is A
A 28-year-old woman with a long-standing history of anorexia nervosa presents with severe back pain and tenderness over the thoracic vertebrae.
-
What is the most likely diagnosis?
- A) Pneumonia
- B) Pulmonary embolism
- C) Pericardial effusion
- D) Compression fracture
- E) Metastatic bone lesions
- Why not A, B, C): These conditions may refer pain to the back but do not cause vertebral tenderness.
- Why not E): No evidence points towards a primary malignancy.
A patient with anorexia nervosa and severe thoracic back pain is suspected to have a compression fracture.
-
What is the initial investigation?
- A) Chest X-ray
- B) MRI of the spine
- C) Bone scintigraphy
- D) CT scan of the spine
- E) Serum calcium levels
- Why not A): Chest X-ray will not adequately assess vertebral fractures.
- Why not C, D): Bone scintigraphy and CT scan are secondary options.
- Why not E): Serum calcium levels are important but not the initial diagnostic test.
A patient with a confirmed compression fracture due to osteoporosis secondary to anorexia nervosa.
-
What is the initial management?
- A) Bed rest and analgesics
- B) Vertebroplasty
- C) Bisphosphonate therapy
- D) Hormone replacement therapy
- E) Physiotherapy
- Why not B, C, D, E): These are secondary treatments once the acute pain is managed.
A patient with anorexia nervosa and osteoporosis is being treated for a compression fracture.
-
What is the best long-term management?
- A) High-calcium diet
- B) Bisphosphonate therapy
- C) Vitamin D supplementation
- D) Weight restoration and nutritional rehabilitation
- E) Hormone replacement therapy
- Why not A, B, C, E): These are adjunctive treatments but not the primary long-term management.
A patient with anorexia nervosa for over a year presents for a routine check-up.
-
What is the best initial screening test for osteoporosis?
- A) Serum calcium levels
- B) Bone mineral density (BMD) test
- C) MRI of the spine
- D) Bone scintigraphy
- E) Serum vitamin D levels
- Why not A, C, D, E): These tests are either not specific for screening or are secondary tests.
A 28-year-old woman with long-standing anorexia nervosa presents with severe thoracic back pain and tenderness.
-
What is the initial investigation?
- A) Chest X-ray
- B) MRI of the spine
- C) Bone scintigraphy
- D) CT scan of the spine
- E) Serum calcium levels
- Why not A): Chest X-ray will not adequately assess vertebral fractures.
- Why not C, D): Bone scintigraphy and CT scan are secondary options.
- Why not E): Serum calcium levels are important but not the initial diagnostic test.
A patient with a confirmed compression fracture due to osteoporosis secondary to anorexia nervosa.
-
What is the initial management?
- A) Bed rest and analgesics
- B) Vertebroplasty
- C) Bisphosphonate therapy
- D) Hormone replacement therapy
- E) Physiotherapy
- Why not B, C, D, E): These are secondary treatments once the acute pain is managed.
A patient continues to have severe pain after initial management of a compression fracture.
-
What is the next step in management?
- A) Increase analgesics
- B) Vertebroplasty
- C) Bisphosphonate therapy
- D) Hormone replacement therapy
- E) Refer to pain specialist
- Why not A): Increasing analgesics may not be sufficient.
- Why not C, D): These are adjunct treatments.
- Why not E): Referral may be considered but vertebroplasty is a direct intervention.
A patient with a history of anorexia nervosa and osteoporosis seeks advice on long-term management.
-
What is the most important long-term management strategy?
- A) High-calcium diet
- B) Bisphosphonate therapy
- C) Vitamin D supplementation
- D) Weight restoration and nutritional rehabilitation
- E) Hormone replacement therapy
- Why not A, B, C, E): These are adjunctive treatments but not the primary focus.
A patient with anorexia nervosa for over a year presents for a routine check-up.
-
What is the best initial screening test for osteoporosis?
- A) Serum calcium levels
- B) Bone mineral density (BMD) test
- C) MRI of the spine
- D) Bone scintigraphy
- E) Serum vitamin D levels
- Why not A, C, D, E): These tests are either not specific for screening or are secondary tests.
Patients with longstanding anorexia nervosa (>6 months) are at significantly increased risk of
pathological fractures due to osteopenia and osteoporosis caused by malnutrition and estrogen
deficiency.
Osteopenia and osteoporosis is a complication of long-standing anorexia nervosa. These changes
are often treated once anorexia nervosa are successfully treated and adequate weight gain is
obtained. Vertebral column is the most common site of pathologic fractures due to osteoporosis.
In this patient with long-standing anorexia nervosa, the pain and tenderness over thoracic vertebrae
makes compression fracture the most likely diagnosis.
Pneumonia (option B), pulmonary embolism (option C), and pericardial effusion (option D) may be
associated with referred pain to the back, but there is be no tenderness.
Metastatic bone lesions (option E) can cause pathologic fractures; however, with the presence of
anorexia nervosa as a significant risk factor on one hand, and no pointers towards an underlying
primary malignancy on the other, metastatic bone disease is less likely.
Which one of the following is a common feature seen in anorexia nervosa?
A. Bradycardia.
B. Normally-developed secondary sexual characteristics.
C. Calloused knuckles.
D. Hirsutism.
E. Heat intolerance.
Correct Answer Is A
Of the options, bradycardia is the most common finding in patient with anorexia nervosa.
(Option B) Sexual characteristics in patients with anorexia nervosa are often absent or
underdeveloped depending on time of the onset of the disease.
(Option C) Calloused knuckles are seen in patients with binge eating and purging subtype of
anorexia nervosa where vomiting induction frequently occurs as a compensatory means. It is not
as common as bradycardia.
(Option D) Hirsutism is not a feature of anorexia nervosa; rather such patients have lanugo (the fine
thin hair covering the body) which is different from hirsutism.
(Option E) Cold intolerance (not heat tolerance) is a feature of anorexia nervosa; however, shivering
in the cold is not seen).
TOPIC REVIEW
Different complications associated with anorexia nervosa are as follows:
Endocrine/metabolic
Menstrual dysfunction
Delayed menarche/ secondary amenorrhea
Delayed growth and development
Regression of secondary sexual characteristics
Decreased metabolic rate and decreased resting energy expenditure
Appetite and thirst dysregulation
Low parathyroid hormone levels
Elevated liver function
Anemia
Hypoglycemia
Decreased white cell count
Protein-calorie malnutrition
Vitamin, mineral and essential fatty acids deficiencies
Ketonuria
Cold intolerance
Depressed immune function
Atrophied breasts
Musculoskeletal
Decreased bone mass
Osteopenia
Stress fractures
Decreased muscle mass
Cardiovascular
Cardiac arrhythmias
ECG abnormalities
Bradycardia (HR<60bpm)
Hypotension
Orthostatic hypotension
Decreased left ventricular mass and stroke volume
Congestive heart failure
Cardiac arrest
Peripheral edema
Gastrointestinal
Parotid and submandibular gland hypertrophy (usually in binge eating/purging type)
Delayed gastric emptying
Constipation/obstipation
Hypoactive bowel sounds
Postprandial discomfort, bloating after small meals
Psychoneurological
Depression
Anxiety
Structural brain abnormalities/brain tissue loss
Impaired concentration
Lack of insight
Sleep disturbances
Dermatological / other
Acrocyanosis
Lanugo – fine soft hair
Xerosis (dry, scaly skin)
Brittle hair
Hair loss
Calloused knuckles (in binge eating/purging type where vomiting induction occurs as a
compensatory behavior)
Eroded enamel (in binge eating/purging type)
Negative signs
Normal fundi or visual fields
No organomegaly
No lymphadenopathy
A 19-year-old woman with a two-year history of anorexia nervosa is brought for assessment. She is extremely thin, weighs 37.5 kg and has a body mass index of 15 kg/m2. Her menses started at 14 years of age, but stopped one year ago. Which one of the following investigations is not indicated
in this patient?
A. Liver function tests (LFTs).
B. Electrocardiogram.
C. Serum gonadotropins (FSH and LH).
D. Electrolytes.
E. DXA scan.
Correct Answer Is C.
The following are the investigations to consider in every patients presenting with anorexia
nervosa:
* Serum electrolytes (option D)
* Blood urea nitrogen
* Serum creatinine
* Serum glucose
* Serum calcium, phosphorous, and magnesium
* Thiamine
* Serum albumin
* Liver function tests (option A) (aspartate aminotransferase, alanine aminotransferase, and alkaline phosphatase)
* Internationalized Normalized Ratio (INR)
* Complete blood count (CBC) including differential
* Erythrocyte sedimentation rate
* Thyroid stimulating hormone (TSH)
* Electrocardiogram (ECG) (option B)
* Urinalysis for specific gravity
A dual energy X-ray absorptiometry (DXA) (option E) scan is also indicated for all patients with
anorexia nervosa of longer than 6 months duration. For this patient with 2-year history of anorexia
nervosa, a DXA scan should be performed to assess the presence of osteopenia or osteoporosis.
An echocardiography should be performed on all patients with anorexia nervosa with body mass
index (BMI) of less than 14 kg/m2.
FSH and LH are not diagnostically valuable where diagnosis of anorexia nervosa is clear based on
the presence of emaciation in conjunction with self-induced starvation and intense fear of weight
gain. In such cases, amenorrhea is the result of the underlying malnutrition and estrogen deficiency
(hypopituitary hypogonadism).
Measurement of FSH/LH is only indicated if the diagnosis of anorexia nervosa cannot be clinched through physical exams and history and other cause of amenorrhea such as ovarian failure have been considered.
NOTE - If the diagnosis of anorexia nervosa is not clear, other causes that can present with weight loss, malabsorption, or amenorrhea should be investigated.
The following conditions are
possible causes to consider:
* Neoplasm
* Chronic infections (e.g. tuberculosis or acquired immunodeficiency syndrome)
* Uncontrolled diabetes mellitus
* Hyperthyroidism
* Malabsorption syndromes (e.g., celiac disease)
* Inflammatory bowel disease
* Pregnancy
* Primary ovarian failure
* Polycystic ovarian syndrome
* Pituitary prolactinoma
A 57-year-old man, who is a chronic alcohol user, is admitted to the hospital after he was found out to have developed chronic liver disease. Three days after the admission, he progressively becomes aggressive and starts to shout at the ward staff and threatening them. Which one of the following is the most appropriate medication to give him?
A. Diazepam.
B. Lorazepam.
C. Haloperidol.
D. Olanzapine.
E. Droperidol.
Correct Answer Is B.
With chronic alcohol use in this patient, the presentation is most likely due to alcohol withdrawal syndrome (AWS). AWS is characterized by a range of signs and symptoms including tremor, sweating, nausea and vomiting, anxiety, agitation, headache, diarrhea, tachycardia, delirium, perceptual disturbances, tachycardia and hypertension. Seizures occasionally occur. In 50%, there is only one episode of seizure.
Clinical presentation begins within the first 6 to 24 hours after last ingestion and typically persists 72 hours, but may last for weeks.When medications are indicated for treatment of AWS, benzodiazepines are first-line. Diazepam 10-20 mg is given orally every 2 hours until symptoms subside or the maximum daily dose of 120mg is reached. Diazepam can be given on subsequent days if symptoms persist.
In patients with severe liver disease, however, a short-acting benzodiazepine without active metabolite such as lorazepam or oxazepam should replace diazepam.
Diazepam is first metabolized by hepatic oxidation resulting in production of active metabolite. The drug then undergoes glucuronidation. In the elderly and patients with liver disease, benzodiazepine oxidation is decreased and can results in accumulation of the drug, resulting in excessive sedation and respiratory suppression.
Lorazepam and oxazepam metabolism is minimally affected by age and liver disease; therefore, these drugs appear to be safest choices among the various benzodiazepines for treating AWS in the elderly and patients with liver disease.
Haloperidol and droperidol are used when psychotic features or agitation persists despite treatment with benzodiazepine.
NOTE – Benzodiazepines and, to a lesser extent, haloperidol and droperidol, may worsen the symptoms of hepatic encephalopathy.
A 73-year-old woman with end-stage renal disease (ESRD), who has been well-controlled on routine
peritoneal dialysis, and has been coping well thus far, suddenly declines to have her dialysis
session today. On examination, she is found to have a temperature of 38.3°C and mild tenderness
in upper abdomen. Which one of the following could be the most likely additional finding in this
patient?
A. Disorientation.
B. Depressed mood.
C. Blunted affect.
D. Delusions.
E. Hypervigilance.
Correct Answer Is A
- Depression is common in patients with ESRD (End-Stage Renal Disease) on dialysis and can lead to treatment withdrawal.
- However, fever and abdominal tenderness suggest an infection, not depression.
- Infections can cause delirium, especially in older adults. For delirium, disorientation is a key symptom.
- Depressed Mood (Option B): Unlikely since the patient has been coping well until now.
- Blunted Affect (Option C), Delusions (Option D), Hypervigilance (Option E): These are symptoms of psychotic disorders, which usually develop gradually, making them less likely in this sudden scenario.
The patient’s fever and abdominal tenderness point towards an infection, possibly causing delirium, rather than depression or a psychotic disorder.
Depression is common among patients with ESRD and on dialysis. It is a major cause of treatment
withdrawal in such patients. However, sudden decline associated with findings of fever and
abdominal tenderness are against depression as the cause of treatment refusal in this patient.
This patient’s fever is a pointer towards infection, as is the abdominal tenderness. Infections may
cause delirium, especially in aging people. For delirium to be the underlying cause to this
presentation, disorientation should be present on examination.
Depressed mood (option B) is an unlikely finding in a patient who has been coping well so far, but
all of the sudden refuses treatment. Blunted affect (option C), delusions (option D) and
hypervigilance (option E) would have been expected in psychotic mental disorders. Such disorders
usually have a more insidious onset making them less likely explanations in this scenario.
A 37-year-old man presents to your practice with complaint of erectile dysfunction. A few months
ago, he found out that her wife had an extramarital affair and developed low mood. After 5 weeks,
he went to see a doctor and was diagnosed with depression and prescribed sertraline. Recently, he
has started a new relationship but is experiencing erectile dysfunction. Which one of the following
would be the most appropriate option for management of his current problem?
A. Stop sertraline.
B. Switch to fluoxetine.
C. Switch to fluvoxamine.
D. Prescribe sildenafil.
E. Prescribe clomipramine.
Correct Answer Is D
Many drugs are capable of producing sexual dysfunction. The reported sexual issues by the
patients include: decreased libido, erectile or ejaculatory problems, and decreased lubrication in
women.
The most important drug classes associated with such problems are: antihypertensives,
antipsychotics, antidepressants and recreational drugs.
Selective serotonin reuptake inhibitors (SSRIs) such as sertraline, fluoxetine, paroxetine,
fluvoxamine, citalopram, etc are all well-known to be associated with sexual dysfunction as one of
their most prominent adverse effects that could lead to noncompliance.
The reported sexual adverse outcomes of SSRIs are:
Decreased libido
Delayed ejaculation (this effect has been used for treatment of premature ejaculation)
Erectile dysfunction
Painful ejaculation
The issue of sexual dysfunction associated with antidepressants is complex. The problem may be
caused by other factors than the medication, while strongly believed otherwise by the patient.
There might be an unrecognizable interaction between the drug and psychological effects. Several
strategies approaches have been proposed
Where the medication is considered drug-induced, several strategies might be tried to reverse the
condition including:
Dose reduction
Switching to another drug from a different drug group
Drug holidays
However, prescribing a phosphodiesterase type 5 inhibitor such as sildenafil or tadalafil in
anticipation of intercourse has become the standard of care for men. In women, these drugs have
shown some benefits such as increased lubrication.
Switching to other SSRIs such as fluoxetine, fluvoxamine, paroxetine, etc is unlikely to resolve the
problem because these drugs are associated with sexual dysfunction as well.
Low-dose clomipramine has proven effective in treatment of premature ejaculation. Recent studies
suggest that it could have an effect in delayed ejaculation; however, sildenafil remains the standard
of care in most patients.
‘NOTE - For patients with depression who develop sexual side effects while on SSRIs, switching to nafazodone, bupropion, or mirtazapine is an alternative. These drugs have minimal, if any, impact
on sexual function.’
A 48-year-old man presents with complaint of significantly delayed ejaculation for the past 2
months. He has the history of major depression, for which he is on sertraline. Four months ago, the
dose of sertraline was increased to 100mg, daily, after he felt that his previously well controlled
symptoms are returning. At present, he is stable and symptom free. Which one of the following
would be the most appropriate management option regarding his ejaculation problem?
A. Switch to fluoxetine.
B. Stop sertraline.
C. Advise sildenafil in anticipation of intercourse.
D. Reduce the dose of sertraline.
E. Prescribe clomipramine.
Correct Answer Is C
SSRIs (Selective Serotonin Reuptake Inhibitors) like sertraline, fluoxetine, paroxetine, fluvoxamine, and citalopram can cause sexual side effects, which may lead to patients stopping their treatment. The common sexual side effects include:
- Decreased libido (sexual desire)
- Delayed ejaculation (used to treat premature ejaculation)
- Erectile dysfunction
- Painful ejaculation
- Drug Holidays: Temporarily stopping the medication.
- Dose Reduction: Lowering the medication dose.
- Switching Antidepressants: Choosing drugs like bupropion, mirtazapine, or nefazodone that have less impact on sexual function.
- Phosphodiesterase Type 5 Inhibitors: Using medications like sildenafil (Viagra) or tadalafil before intercourse.
- Cognitive Behavioral Therapy: Psychological treatment to manage the effects.
Using a phosphodiesterase type 5 inhibitor, like sildenafil before intercourse, is the standard approach.
- Option A: Switching to other SSRIs (e.g., fluoxetine, fluvoxamine, paroxetine) is not effective because they also cause sexual dysfunction.
- Option B: Stopping sertraline can cause the depression to return.
- Option D: Lowering the sertraline dose in someone who just responded well to an increased dose can cause a relapse in depression.
- Option E: Adding clomipramine, a tricyclic antidepressant used for premature ejaculation, can worsen delayed ejaculation.
In summary, managing sexual side effects of SSRIs involves careful consideration of alternative treatments and sometimes using medications like sildenafil to help with the issue.
Selective serotonin reuptake inhibitors (SSRIs) such as sertraline, fluoxetine, paroxetine,
fluvoxamine, citalopram, etc are all well-known to be associated with sexual dysfunction as one of
their most prominent adverse effects that could lead to noncompliance to the treatment from the
patients.
The reported sexual adverse outcomes of SSRIs are:
Decreased libido
Delayed ejaculation (this effect has been used for treatment of premature ejaculation)
Erectile dysfunction
Painful ejaculation
Strategies have been proposed for dealing with sexual adverse effects of SSRIs, including:
Drug holidays
Dose reduction
Switching to other antidepressant with minimal or no impact on sexual functioning such as
bupropion, mirtazapine, or nafazodone
Use of phosphodiesterase type 5 inhibitors such as sildenafil or tadalafil
Cognitive behavioral therapy
Of these options, however, using a phosphodiesterase type 5 inhibitor such as sildenafil before
intercourse has become the standard of care.
(Option A) Switching to other SSRIs such as fluoxetine, fluvoxamine, paroxetine, etc is unlikely to
resolve the problem because these drugs are associated with sexual dysfunction as well.
(Option B) Cessation of sertraline will result in relapse of the recently-controlled depression.
(Option D) Decreasing the current dose of sertraline in a patient who has just responded the
incremental dose increase due to uncontrolled depressive symptoms is very likely to result in
relapse; therefore, not an appropriate option.
(Option E) Clomipramine is a tricyclic antidepressant used for treatment of premature ejaculation.
Addition of clomipramine will deteriorate delayed ejaculation.
A 24-year-old woman presents with elevated mood, pressured speech, agitation and flight of ideas over the past 2 weeks. There is no history of drug abuse in the past. The patient is fully cooperative. Which one of the following is the drug of choice in this situation?
A. Sodium valproate.
B. Lithium.
C. Carbamazepine.
D. Haloperidol.
E. Quetiapine.
Correct Answer Is B.
The clinical history of this patient suggests acute mania, likely due to bipolar I disorder. Bipolar I disorder involves mood disturbances with manic symptoms lasting at least one week. These symptoms significantly impact daily functioning.
- Elevated Mood: Persistent and abnormally elevated mood for at least one week
- Increased Self-Esteem: Grandiosity
- Distractibility
- Excessive Activities: Increased involvement in various activities
- Talkativeness: More talkative than usual, often with pressured speech
- Psychomotor Agitation
- Flights of Ideas
- Increased Sexual Activity
- Increased Goal-Directed Activities
- Lithium: First choice for both acute treatment and maintenance therapy.
- Antipsychotics: Olanzapine, quetiapine (option E), risperidone, and aripiprazole can be used alone or with lithium for acute mania.
- Sodium Valproate (option A): Used with atypical antipsychotics as an alternative for some patients.
- Haloperidol (option D): Used for sedation in patients with acute mania. Intramuscular haloperidol is the best initial management for non-cooperative and acutely agitated patients.
- Carbamazepine (option C): A mood stabilizer, but less effective than lithium or atypical antipsychotics for acute mania in bipolar disorder.
In summary, lithium is the primary treatment, with antipsychotics and other medications used based on the patient’s specific needs.
The clinical history of this patient is consistent with acute mania, probably due to bipolar I disorder.
Bipolar I disorder is a mood disturbance characterized by manic symptoms of at least one week duration. The symptoms should be significant enough to cause considerable impairment or distress in the level of functioning.
Characteristic features of bipolar disorder include:
* Persistent abnormally elevated mood lasting for at least one week
* Increased self-esteem and grandiosity
* Distractibility
* Excessive involvement in activities
* More talkative than usual – pressured speech is often noticed on examination
* Psychomotor agitation
* Flights of ideas
* Increased sexual activity
* Increase in goal-directed activities
Of the given options, lithium is the drug of choice for both acute treatment and maintenace therapy.
Antipsychotics such as olanzapine, quetiapine (option E), risperidone and aripiprazole can be used in combination with lithium or alone for acute mania. Sodium valproate (option A) in combination with atypical antipsychotics is an alternative in specific patients.
Haloperidole (option D) is used for patients with acute manic features, who need sedation. If the patient is not cooperative and acutely agitated, intramuscular haloperidol would be the best initial management. Carbamazepine (option C) and some other anticonvulsants has been used as a mood-stabilzer but evidence shows it is less effective than lithium or atypical antipsychotics in management of acute mania in bipolar disorder.
1.
A 32-year-old is brought to the emergency department after she was arrested by the police in a
shopping mall. She is dressed in colorful dresses. According to the police, she was shouting at
everyone in the mall. She was claiming in an excited manner that her mother was virgin and
married God. In the emergency department and during the interview she keeps saying that she
hears the voice of God, and that she is planning to become pregnant from him. Which one of the
following is the most likely cause to her presentation?
A. Antisocial personality disorder.
B. Opium abuse.
C. Alcohol intoxication.
D. Schizophrenia.
E. Borderline personality disorder.
Correct Answer Is D
Two points are of diagnostic significance in this case scenario. First is the delusional thought
content evident by strongly-held bizarre false beliefs that the patient’s mother married God, and
that she is planning to be conceived by him. Second is the presence of auditory hallucinations.
These two together makes a psychotic episode the most likely diagnosis. Of the given option, the
only one that can represent these is schizophrenia.
(Option A) Antisocial personality disorder is characterized by continuous antisocial or criminal
acts, inability to conform to social rules, impulsivity, disregard for the rights of others,
aggressiveness, lack of remorse, and deceitfulness. There are no delusions or hallucinations,
unless caused by another reason.
(Option B) Opium abuse is not associated with delusions or hallucinations.
(Option C) Acute alcohol intoxication presents varying manifestations depending on the severity,
and can include slurred speech, nystagmus, disinhibited behavior, incoordination, unsteady gait,
memory impairment, stupor or even coma. Hallucinations and delusions are not typical
presentations.
(Option E) Borderline personality disorder is often characterised by an unstable affect, mood
swings, marked impulsivity, unstable relationships, recurrent suicidal behaviours and self-inflicted
injures, chronic feelings of emptiness and boredom, identity disturbances, and inappropriate anger.
If they are stressed they may become psychotic. They use splitting as their main defence
mechanism.